Sie sind auf Seite 1von 59

lOMoARcPSD|4061633

Tb05 - Test bank chapter 5

Financial Accounting (University of Ottawa)

StuDocu is not sponsored or endorsed by any college or university


Downloaded by Rnz Rnz (renzalconera@gmail.com)
lOMoARcPSD|4061633

CHAPTER 5

MERCHANDISING OPERATIONS
SUMMARY OF QUESTION TYPES BY STUDY OBJECTIVE AND LEVEL
OF DIFFICULTY
Item SO LOD Item SO LOD Item SO LOD Item SO LOD Item SO LOD
True-False Statements
1. 1 E 13. 2 E 25. 3 M 37. 4 M *49. 6 M
2. 1 E 14. 2 E 26. 3 E 38. 4 M *50. 6 E
3. 1 E 15. 2 E 27. 3 E 39. 4 E *51. 6 M
4. 1 E 16. 2 E 28. 3 E 40. 5 E *52. 6 E
5. 1 E 17. 2 E 29. 3 M 41. 5 M *53. 6 M
6. 1 E 18. 2 E 30. 3 M 42. 5 M *54. 6 E
7. 1 M 19. 2 M 31. 4 M 43. 5 E *55. 6 E
8. 1 E 20. 2 M 32. 4 E 44. 5 E *56. 6 M
9. 1 M 21. 3 E 33. 4 E 45. 5 E *57. 6 M
10. 2 M 22. 3 E 34. 4 E 46. 5 E
11. 2 M 23. 3 M 35. 4 E 47. 5 M
12. 2 M 24. 3 M 36. 4 M *48. 6 E
Multiple Choice Questions
58. 1 E 79. 1,6 E 100. 3 E 121. 4 E 142. 5 E
59. 1 E 80. 2 E 101. 3 E 122. 4 E 143. 5 M
60. 1 E 81. 2 M 102. 3 M 123. 4 E 144. 5 M
61. 1 M 82. 2 E 103. 3 E 124. 4 H *145. 6 M
62. 1 M 83. 2 M 104. 3 M 125. 4 M *146. 6 E
63. 1 E 84. 2 M 105. 3 E 126. 4 E *147. 6 E
64. 1 M 85. 2 E 106. 3 E 127. 4 E *148. 6 E
65. 1 E 86. 2 M 107. 3 M 128. 4 M *149. 6 E
66. 1 E 87. 2 E 108. 3 E 129. 4 E *150. 6 E
67. 1 E 88. 2 E 109. 3 M 130. 4 M *151. 6 M
68. 1 M 89. 2 E 110. 3 M 131. 4 E *152. 6 M
69. 1 M 90. 3 E 111. 3 E 132. 4 M *153. 6 E
70. 1 E 91. 3 M 112. 3 E 133. 4 M *154. 6 E
71. 1 E 92. 3 E 113. 3 E 134. 5 M *155. 6 E
72. 1 E 93. 3 E 114. 3 E 135. 5 M *156. 6 M
73. 1 E 94. 3 E 115. 4 E 136. 5 E *157. 6 M
74. 1 M 95. 3 E 116. 4 E 137. 5 M *158. 6 M
75. 1 E 96. 3 M 117. 4 E 138. 5 E *159. 6 M
76. 1 E 97. 3 E 118. 4 E 139. 5 M *160. 6 M
77. 1,6 M 98. 3 E 119. 4 E 140. 5 E *161. 6 M
78. 1,6 E 99. 3 M 120. 4 M 141. 5 E

Note: E = Easy M = Medium H = Hard

*This topic is dealt with in an Appendix to the chapter.

Copyright © 2014 John Wiley & Sons Canada, Ltd. Unauthorized copying, distribution, or transmission of this page is prohibited

Downloaded by Rnz Rnz (renzalconera@gmail.com)


lOMoARcPSD|4061633

5-2 Test Bank for Financial Accounting: Tools for Business Decision-Making, 6th Canadian Edition

SUMMARY OF QUESTION TYPES BY STUDY OBJECTIVE AND LEVEL


OF DIFFICULTY (CONTINUED)
Item SO LOD Item SO LOD Item SO LOD Item SO LOD Item SO LOD
Exercises
162. 2 H 168. 2,3,6 E 174. 4,5 E *180. 6 H *186. 6 E
163. 2,3 E 169. 3 M 175. 4,5 E *181. 6 M *187. 6 M
164. 2,3 M 170. 3 E 176. 4,5 E *182. 6 E *188. 6 E
165. 2,3 E 171. 4 E 177. 4,5 E *183. 6 E
166. 2,3 E 172. 4 E 178. 5 M *184. 6 E
167. 2,3 H 173. 4 M 179. 5,6 H *185. 6 M
Matching
189. 1–3,5,6 E,M
Short-Answer Essay
190. 1 E 193. 4 E 196. 4 E 199. 5 E
191. 1,2,6 M 194. 4 E 197. 4 M *200. 6 E
192. 1,6 E 195. 4 M 198. 4,5 M

Note: E = Easy M = Medium H = Hard

*This topic is dealt with in an appendix to the chapter.

Copyright © 2014 John Wiley & Sons Canada, Ltd. Unauthorized copying, distribution, or transmission of this page is prohibited

Downloaded by Rnz Rnz (renzalconera@gmail.com)


lOMoARcPSD|4061633

5-3 Merchandising Operations

SUMMARY OF STUDY OBJECTIVES BY QUESTION TYPE


Item Type Item Type Item Type Item Type Item Type Item Type Item Type
Study Objective 1
1. TF 6. TF 59. MC 64. MC 69. MC 74. MC 79. MC
2. TF 7. TF 60. MC 65. MC 70. MC 75. MC 189. Ma
3. TF 8. TF 61. MC 66. MC 71. MC 76. MC 190. SAE
4. TF 9. TF 62. MC 67. MC 72. MC 77. MC 191. SAE
5. TF 58. MC 63. MC 68. MC 73. MC 78. MC 192. SAE
Study Objective 2
10. TF 15. TF 20. TF 84. MC 89. MC 166. Ex
11. TF 16. TF 80. MC 85. MC 162. Ex 167. Ex
12. TF 17. TF 81. MC 86. MC 163. Ex 168. Ex
13. TF 18. TF 82. MC 87. MC 164. Ex 189. Ma
14. TF 19. TF 83. MC 88. MC 165. Ex 191. SAE
Study Objective 3
21. TF 28. TF 94. MC 101. MC 108. MC 163. Ex 170. Ex
22. TF 29. TF 95. MC 102. MC 109. MC 164. Ex 189. Ma
23. TF 30. TF 96. MC 103. MC 110. MC 165. Ex
24. TF 90. MC 97. MC 104. MC 111. MC 166. Ex
25. TF 91. MC 98. MC 105. MC 112. MC 167. Ex
26. TF 92. MC 99. MC 106. MC 113. MC 168. Ex
27. TF 93. MC 100. MC 107. MC 114. MC 169. Ex
Study Objective 4
31. TF 37. TF 118. MC 124. MC 130. MC 173. Ex 194. SAE
32. TF 38. TF 119. MC 125. MC 131. MC 174. Ex 195. SAE
33. TF 39. TF 120. MC 126. MC 132. MC 175. Ex 196. SAE
34. TF 115. MC 121. MC 127. MC 133. MC 176. Ex 197. SAE
35. TF 116. MC 122. MC 128. MC 171. Ex 177. Ex 198. SAE
36. TF 117. MC 123. MC 129. MC 172. Ex 193. SAE
Study Objective 5
40. TF 44. TF 134. MC 138. MC 142. MC 175. Ex 179. Ex
41. TF 45. TF 135. MC 139. MC 143. MC 176. Ex 189. Ma
42. TF 46. TF 136. MC 140. MC 144. MC 177. Ex 198. SAE
43. TF 47. TF 137. MC 141. MC 174. Ex 178. Ex 199. SAE
*Study Objective 6
*48. TF *55. TF *146. MC *153. MC *160. MC *183. Ex *191. SAE
*49. TF *56. TF *147. MC *154. MC *161. MC *184. Ex *192. SAE
*50. TF *57. TF *148. MC *155. MC *168. Ex *185. Ex *200. SAE
*51. TF *77. MC *149. MC *156. MC *179. Ex *186. Ex
*52. TF *78. MC *150. MC *157. MC *180. Ex *187. Ex
*53. TF *79. MC *151. MC *158. MC *181. Ex *188. Ex
*54. TF *145. MC *152. MC *159. MC *182. Ex *189. Ma
Note: TF = True/False Ma = Matching
MC = Multiple Choice Ex = Exercise SAE = Short Answer Essay

*This topic is dealt with in an Appendix to the chapter.

Copyright © 2014 John Wiley & Sons Canada, Ltd. Unauthorized copying, distribution, or transmission of this page is prohibited

Downloaded by Rnz Rnz (renzalconera@gmail.com)


lOMoARcPSD|4061633

5-4 Test Bank for Financial Accounting: Tools for Business Decision-Making, 6th Canadian Edition

CHAPTER STUDY OBJECTIVES

1. Identify the differences between service and merchandising companies. A service


company performs services. It has service or fee revenue and operating expenses. A
merchandising company sells goods. It has sales revenue, cost of goods sold, and gross
profit in addition to operating expenses. Both types of company may also report non-
operating items and each would report income tax expense.

2. Prepare entries for purchases under a perpetual inventory system. The Merchandise
Inventory account is debited for all purchases of merchandise and for freight costs if those
costs are paid by the buyer (shipping terms FOB shipping point). It is credited for purchase
discounts, and purchase returns and allowances.

3. Prepare entries for sales under a perpetual inventory system. When inventory is sold,
two entries are required: (1) Cash or Accounts Receivable is debited and Sales is credited
for the selling price of the merchandise, and (2) Cost of Goods Sold is debited and
Merchandise Inventory is credited for the cost of inventory items sold. Contra revenue
accounts are used to record sales returns and allowances and sales discounts. Two journal
entries are also required for sales returns so that both the selling price and the cost of the
returned merchandise are recorded. Freight costs paid by the seller (shipping terms FOB
destination) are recorded as an operating expense.

4. Prepare a single-step and a multiple-step income statement. In a single-step income


statement, all data (except for income tax expense) are classified under two categories—
revenues or expenses—and profit before income tax is determined in one step. Income tax
expense is separated from the other expenses and reported separately after profit before
income tax to determine profit (loss).
A multiple-step income statement shows several steps in determining profit. Step 1 deducts
sales returns and allowances and sales discounts from gross sales to determine net sales.
Step 2 deducts the cost of goods sold from net sales to determine gross profit. Step 3
deducts operating expenses (which can be classified by nature or by function) from gross
profit to determine profit from operations. Step 4 adds or deducts any non-operating items
to determine profit before income tax. Finally, step 5 deducts income tax expense to
determine profit (loss).

5. Calculate the gross profit margin and profit margin. The gross profit margin, calculated
by dividing gross profit by net sales, measures the gross profit earned for each dollar of
sales. The profit margin, calculated by dividing profit by net sales, measures the profit
earned for each dollar of sales. Both are measures of profitability that are closely watched
by management and other interested parties.

*6. Prepare entries for purchases and sales under a periodic inventory system and
calculate cost of goods sold (Appendix 5A). The periodic inventory system differs from
the perpetual inventory system in that separate temporary accounts are used in the periodic

Copyright © 2014 John Wiley & Sons Canada, Ltd. Unauthorized copying, distribution, or transmission of this page is prohibited

Downloaded by Rnz Rnz (renzalconera@gmail.com)


lOMoARcPSD|4061633

5-5 Merchandising Operations

system to record (1) purchases, (2) purchase returns and allowances, (3) purchase
discounts, and (4) freight costs that are paid by the buyer (shipping terms FOB shipping
point). The formula for cost of goods purchased is as follows: Purchases – purchase returns
and allowances – purchase discounts = net purchases; and net purchases + freight in =
cost of goods purchased.
Both systems use temporary accounts to record (1) sales, (2) sales returns and allowances,
and (3) sales discounts. However, in a periodic inventory system, only one journal entry is
made to record a sale of merchandise as the cost of goods sold is not recorded throughout
the period. Instead, the cost of goods sold is determined at the end of the period.
To determine the cost of goods sold, first calculate the cost of goods purchased, as
indicated above. Then, calculate the cost of goods sold as follows: Beginning inventory +
cost of goods purchased = cost of goods available for sale; and cost of goods available for
sale – ending inventory = cost of goods sold.
At the end of the period, the Merchandise Inventory account is adjusted to reflect its proper
balance as determined from the inventory count results. The change in this account is
allocated to the Cost of Goods Sold account as are the balances in the Freight In and
Purchases account and any related contra accounts.

Copyright © 2014 John Wiley & Sons Canada, Ltd. Unauthorized copying, distribution, or transmission of this page is prohibited

Downloaded by Rnz Rnz (renzalconera@gmail.com)


lOMoARcPSD|4061633

5-6 Test Bank for Financial Accounting: Tools for Business Decision-Making, 6th Canadian Edition

TRUE-FALSE STATEMENTS

1. A physical inventory count should be done at least once a year regardless of whether a
perpetual or periodic inventory system is being used.

2. The operating cycle of a merchandising company is generally shorter than that of a service
company.

3. Sales less operating expenses equal gross profit.

4. Under a perpetual inventory system, the cost of goods sold is determined each time a sale
occurs.

5. Inventory is usually the largest current asset for a merchandiser.

6. Cost of Goods Sold is considered an operating expense for a merchandising company.

7. Operating expenses are subtracted from revenue for a service company and from gross profit
for a merchandising company.

8. Gross sales less cost of goods sold is called gross profit.

9. Cost of goods available for sale is considered an operating expense for a merchandising
company.

10. When the terms of sale are FOB shipping point, the seller is responsible for any damages to
the goods during shipping.

11. Freight terms will specify the point at which ownership of the goods is transferred from the
seller to the buyer.

12. Under a perpetual inventory system, the freight costs incurred by the buyer to acquire the
inventory are added to the Merchandise Inventory account.

13. Under the perpetual inventory system, purchases of merchandise for sale are recorded in
the Merchandise Inventory account.

Copyright © 2014 John Wiley & Sons Canada, Ltd. Unauthorized copying, distribution, or transmission of this page is prohibited

Downloaded by Rnz Rnz (renzalconera@gmail.com)


lOMoARcPSD|4061633

5-7 Merchandising Operations

14. Freight costs incurred on incoming merchandise are an operating expense to the buyer.

15. The terms 2/10, n/30 mean that a 2% discount is allowed on payments made over 10 days
but within the credit period.

16. Discounts taken for early payment of an invoice are called sales discounts by the buyer.

17. If merchandise costing $2,500, terms 2/10 n/30, is paid within 10 days, the amount of the
purchase discount is $250.

18. Under the perpetual inventory system, a discount taken for early payment is credited to the
Merchandise Inventory account.

19. A quantity discount is recorded separately, the same way as a purchase discount.

20. If a quantity discount of 10% is received on a purchase of $10,000, inventory would be


recorded at $9,000.

21. Sales revenues are earned when the goods are transferred from buyer to seller.

22. Sales revenues are recorded by the seller when an order is placed by a buyer.

23. The Sales Returns and Allowances account and the Sales Discounts account are both
classified as expense accounts.

24. When goods are shipped with terms of FOB shipping point, freight costs will be booked as
an operating expense for the seller.

25. Sales Allowances and Sales Discounts are both designed to encourage customers to pay
their accounts promptly.

26. Sales Discounts is a contra revenue account to Sales.

27. The normal balance of Sales Returns and Allowances is a credit.

28. When the terms of sale include a sales discount, it usually is advisable for the buyer to pay

Copyright © 2014 John Wiley & Sons Canada, Ltd. Unauthorized copying, distribution, or transmission of this page is prohibited

Downloaded by Rnz Rnz (renzalconera@gmail.com)


lOMoARcPSD|4061633

5-8 Test Bank for Financial Accounting: Tools for Business Decision-Making, 6th Canadian Edition

within the discount period.

29. Merchandise is sold for $2,500 with terms 1/10, n/30. If $500 of the merchandise is returned
prior to payment and the invoice is paid within the discount period, the amount of the sales
discount is $20.

30. When returned merchandise is defective, the seller’s sales account is debited.

31. The multiple-step income statement is considered more useful than the single-step income
statement for a merchandising company because it highlights the components of profit.

32. Operating expenses are similar in merchandising and service companies.

33. Gross profit appears on both the single-step and multiple-step forms of the income
statement.

34. Non-operating activities include revenues and expenses that are related to the company’s
main operations.

35. Corporations following IFRS must classify their expenses either by nature or by function.

36. Profit from operations appears on both the single-step and multiple-step forms of the income
statement.

37. A merchandising company’s profit from operations is determined by subtracting cost of


goods sold from net sales.

38. Interest revenue for a merchandising company is usually reported in the non-operating
activities section of the income statement.

39. Companies following ASPE may classify their expenses by nature, but not by function.

40. Gross profit is a measure of the overall profit of a company.

41. Gross profit is expressed as a percentage of gross sales.

42. Gross profit margin is the same as the gross profit amount.

Copyright © 2014 John Wiley & Sons Canada, Ltd. Unauthorized copying, distribution, or transmission of this page is prohibited

Downloaded by Rnz Rnz (renzalconera@gmail.com)


lOMoARcPSD|4061633

5-9 Merchandising Operations

43. If net sales are $1,000,000 and cost of goods sold is $800,000, the gross profit margin is
20%.

44. The gross profit amount is generally considered to be more informative than the gross profit
margin.

45. Gross profit margin is calculated by dividing cost of goods sold by net sales.

46. Profit margin indicates whether a company is controlling operating expenses relative to
sales.

47. Profit margin is calculated by dividing profit by net sales.

*48. Under a periodic inventory system, purchase of inventory is debited to the Purchases
account.

*49. Under a periodic inventory system, freight costs incurred on merchandise purchases by the
buyer should be debited to the Merchandise Inventory account.

*50. Under a periodic inventory system, purchases of merchandise are usually credited to the
Purchases account.

*51. Under a periodic inventory system, freight incurred on merchandise sales by the seller should
be debited to the Freight In account.

*52. Purchase Returns and Allowances and Purchase Discounts are contra expense accounts
with normal credit balances.

*53. Freight In is subtracted from the Purchases account to arrive at cost of goods purchased.

*54. A key difference between the periodic and perpetual inventory systems is the timing of the
calculation of cost of goods sold.

*55. The cost of goods sold section of an income statement prepared under a periodic inventory
system will contain more detail than under a perpetual inventory system.

Copyright © 2014 John Wiley & Sons Canada, Ltd. Unauthorized copying, distribution, or transmission of this page is prohibited

Downloaded by Rnz Rnz (renzalconera@gmail.com)


lOMoARcPSD|4061633

5 - 10 Test Bank for Financial Accounting: Tools for Business Decision-Making, 6th Canadian Edition

*56. On the income statement for a company using the periodic inventory system, the inventory
at the beginning of the period is added to the cost of merchandise purchased for the period to
calculate the cost of goods available for sale during the period.

*57. Compared to a perpetual inventory system, the use of the periodic inventory system will
result in a different value for inventory on the statement of financial position.

Copyright © 2014 John Wiley & Sons Canada, Ltd. Unauthorized copying, distribution, or transmission of this page is prohibited

Downloaded by Rnz Rnz (renzalconera@gmail.com)


lOMoARcPSD|4061633

5 - 11 Merchandising Operations

ANSWERS TO TRUE-FALSE STATEMENTS

Item Ans. Item Ans. Item Ans. Item Ans. Item Ans.
1. T 13. T 25. F 37. F *49. F
2. F 14. F 26. T 38. T *50. F
3. F 15. F 27. F 39. F *51. F
4. T 16. F 28. T 40. F *52. T
5. T 17. F 29. T 41. F *53. F
6. F 18. T 30. F 42. F *54. T
7. T 19. F 31. T 43. T *55. T
8. F 20. T 32. T 44. F *56. T
9. F 21. F 33. F 45. F *57. F
10. F 22. F 34. F 46. T
11. T 23. F 35. T 47. T
12. T 24. F 36. F *48. T

Copyright © 2014 John Wiley & Sons Canada, Ltd. Unauthorized copying, distribution, or transmission of this page is prohibited

Downloaded by Rnz Rnz (renzalconera@gmail.com)


lOMoARcPSD|4061633

5 - 12 Test Bank for Financial Accounting: Tools for Business Decision-Making, 6th Canadian Edition

MULTIPLE CHOICE QUESTIONS

58. The time it takes to go from cash to cash in producing revenues is called the
(a) accounting cycle.
(b) purchasing cycle.
(c) operating cycle.
(d) merchandising cycle.

59. Gross profit equals the difference between net sales and
(a) profit.
(b) cost of goods sold.
(c) operating expenses.
(d) cost of goods sold plus operating expenses.

60. Each of the following companies is a merchandising company except a


(a) wholesale parts company.
(b) candy store.
(c) moving company.
(d) furniture store.

61. Operating profit will result if gross profit exceeds


(a) cost of goods sold.
(b) operating expenses.
(c) purchases.
(d) cost of goods sold plus operating expenses.

62. A merchandiser will have profit from operations of exactly $0 when


(a) net sales equals cost of goods sold.
(b) cost of goods sold equals gross profit.
(c) operating expenses equal net sales.
(d) gross profit equals operating expenses.

63. The largest current asset for a merchandiser is usually


(a) inventory.
(b) prepaid expenses.
(c) cash.
(d) accounts receivable.

64. The primary source of revenue for a wholesaler is generated by


(a) investments.
(b) providing services.
(c) the sale of merchandise.
(d) the sale of property, plant, and equipment the company owns.

Copyright © 2014 John Wiley & Sons Canada, Ltd. Unauthorized copying, distribution, or transmission of this page is prohibited

Downloaded by Rnz Rnz (renzalconera@gmail.com)


lOMoARcPSD|4061633

5 - 13 Merchandising Operations

65. Generally, the revenue account for a merchandising company is called


(a) Sales Revenue or Sales.
(b) Investment Revenue.
(c) Gross Profit.
(d) Net Sales.

66. The operating cycle of a merchandising company is


(a) always one year in length.
(b) generally longer than that of a service company.
(c) about the same as that of a service company.
(d) generally shorter than that of a service company.

67. Net sales less cost of goods sold is called


(a) gross profit.
(b) cost of goods sold.
(c) profit.
(d) profit before income taxes.

68. After gross profit is calculated, operating expenses are deducted to determine
(a) gross margin.
(b) profit (loss) before income tax.
(c) cost of goods sold.
(d) profit margin.

69. Which of the following “formulas” is incorrect?


(a) Gross profit – operating expenses = profit before income tax.
(b) Net sales – cost of goods sold = gross profit.
(c) Net sales – gross profit = cost of goods sold.
(d) Operating expenses – cost of goods sold = gross profit.

70. Beginning inventory plus purchases equals


(a) cost of goods available for sale.
(b) cost of goods sold.
(c) ending inventory.
(d) total inventory on hand.

71. Which of the following is true about inventory systems?


(a) Periodic inventory systems require more detailed inventory records.
(b) Perpetual inventory systems require more detailed inventory records.
(c) A periodic system requires cost of goods sold to be recorded after each sale.
(d) A perpetual system determines cost of goods sold only at the end of the accounting period.

Copyright © 2014 John Wiley & Sons Canada, Ltd. Unauthorized copying, distribution, or transmission of this page is prohibited

Downloaded by Rnz Rnz (renzalconera@gmail.com)


lOMoARcPSD|4061633

5 - 14 Test Bank for Financial Accounting: Tools for Business Decision-Making, 6th Canadian Edition

72. In a perpetual inventory system, cost of goods sold is recorded


(a) on a daily basis.
(b) on a monthly basis.
(c) on an annual basis.
(d) each time a sale occurs.

73. The primary difference between a periodic and a perpetual inventory system is that a
periodic system
(a) keeps a detailed record showing the inventory on hand at all times.
(b) provides better control over inventories.
(c) records the cost of goods sold on the date the sale is made.
(d) determines the cost of goods sold at the end of the accounting period.

74. The physical inventory count is used to determine


(a) cost of inventory purchased during the period.
(b) cost of inventory sold during the period.
(c) the cost of inventory on hand.
(d) the cost of goods available for sale.

75. Inventory becomes part of the cost of goods sold when a company
(a) pays for the inventory.
(b) purchases the inventory.
(c) sells the inventory.
(d) receives payment from the customer.

76. If a company determines cost of goods sold each time a sale occurs, it
(a) must have a computerized accounting system.
(b) uses a combination of the perpetual and periodic inventory systems.
(c) uses a periodic inventory system.
(d) uses a perpetual inventory system.

77. Under a perpetual inventory system


(a) there is no need for a year-end physical count.
(b) increases in inventory resulting from purchases are debited to Purchases.
(c) accounting records continuously disclose the amount of inventory.
(d) the account Purchase Returns and Allowances is credited when goods are returned to
vendors.

78. Under a perpetual inventory system, the following is determined each time a sale occurs:
(a) Gross Profit.
(b) Cost of Goods Sold.
(c) Purchases.
(d) Accounts Receivable.

Copyright © 2014 John Wiley & Sons Canada, Ltd. Unauthorized copying, distribution, or transmission of this page is prohibited

Downloaded by Rnz Rnz (renzalconera@gmail.com)


lOMoARcPSD|4061633

5 - 15 Merchandising Operations

79. Under the perpetual inventory system, which of the following accounts would not be used?
(a) Sales
(b) Purchases
(c) Cost of Goods Sold
(d) Merchandise Inventory

80. The abbreviation "FOB" stands for


(a) free on board.
(b) freight on board.
(c) free only (to) buyer.
(d) freight charge on buyer.

81. On July 10, Swant Inc. purchased $1,000 of inventory on terms of 2/10, n/45. If Swant is
paid for the inventory on August 25th, what is the amount due on that day?
(a) $1,020.
(b) $1,000.
(c) $980.
(d) $990.

82. Under a perpetual inventory system, purchase of inventory is recorded as a debit to the
(a) Supplies account.
(b) Purchases account.
(c) Merchandise Inventory account.
(d) Cost of Goods Sold account.

83. The journal entry by the buyer to record a return of merchandise purchased on account
under a perpetual inventory system would credit
(a) Accounts Payable.
(b) Purchase Returns and Allowances.
(c) Sales.
(d) Merchandise Inventory.

84. A company using a perpetual inventory system that returns goods purchased on credit
would
(a) debit Accounts Payable and credit Merchandise Inventory.
(b) debit Sales and credit Accounts Payable.
(c) debit Cash and credit Accounts Payable.
(d) debit Accounts Payable and credit Purchases.

85. If a purchaser using a perpetual inventory system pays freight costs, then the
(a) Merchandise Inventory account is increased.
(b) Merchandise Inventory account is not affected.
(c) Freight Out account is increased.
(d) Freight In account is increased.

Copyright © 2014 John Wiley & Sons Canada, Ltd. Unauthorized copying, distribution, or transmission of this page is prohibited

Downloaded by Rnz Rnz (renzalconera@gmail.com)


lOMoARcPSD|4061633

5 - 16 Test Bank for Financial Accounting: Tools for Business Decision-Making, 6th Canadian Edition

86. Freight costs incurred by a seller on merchandise sold to customers will cause an increase
(a) in the selling expenses of the buyer.
(b) in operating expenses for the seller.
(c) to the cost of goods sold of the seller.
(d) to a contra revenue account of the seller.

87. Cashmere Corporation purchased merchandise inventory with an invoice price of $16,000
and credit terms of 2/10, n/30. How much cash will Cashmere pay if they pay within the discount
period?
(a) $16,000
(b) $15,680
(c) $14,720
(d) $14,400

88. For a company using a perpetual inventory system, the journal entry to record the purchase
of $3,500 of goods on account, with terms of 4/10, n/30, would include a
(a) debit to accounts payable of $3,500.
(b) credit to accounts payable of $3,360.
(c) debit to merchandise inventory of $3,360.
(d) debit to merchandise inventory of $3,500.

89. A purchase invoice is a document that


(a) provides support for goods sold for cash.
(b) provides evidence of operating expenses incurred.
(c) provides evidence of credit purchases.
(d) serves only as a customer receipt.

90. Under the perpetual inventory system, in addition to making the entry to record the sale, the
seller would
(a) debit Merchandise Inventory and credit Cost of Goods Sold.
(b) debit Cost of Goods Sold and credit Purchases.
(c) debit Cost of Goods Sold and credit Merchandise Inventory.
(d) make no additional entry until the end of the period.

91. Sales revenues are usually considered earned when


(a) cash is received from credit sales.
(b) an order is received.
(c) goods have been transferred from the seller to the buyer.
(d) adjusting entries are made.

92. Sales Discounts is a(n)


(a) contra revenue account.
(b) contra asset account.
(c) revenue account.

Copyright © 2014 John Wiley & Sons Canada, Ltd. Unauthorized copying, distribution, or transmission of this page is prohibited

Downloaded by Rnz Rnz (renzalconera@gmail.com)


lOMoARcPSD|4061633

5 - 17 Merchandising Operations

(d) expense account.

93. Evidence of cash sales is usually supported by


(a) purchase invoices.
(b) sales invoices.
(c) purchase orders.
(d) cash register tapes.

94. Gross sales less sales returns and allowances less sales discounts equals
(a) collectible sales.
(b) net sales.
(c) total sales.
(d) operating sales.

95. The entry to record a sale of $525 with terms of 2/10, n/30 will include a
(a) debit to Sales Discounts for $10.50.
(b) debit to Sales for $514.50.
(c) credit to Accounts Receivable for $525.
(d) credit to Sales for $525.

96. A sales invoice is prepared when goods


(a) are sold for cash.
(b) are sold on credit.
(c) sold on credit are returned.
(d) are sold on credit or for cash.

97. Sales Returns and Allowances is a(n)


(a) asset account.
(b) contra asset account.
(c) expense account.
(d) contra revenue account.

98. The entry to record the return of goods from a customer would include a
(a) debit to Sales.
(b) credit to Sales.
(c) debit to Sales Returns and Allowances.
(d) credit to Sales Returns and Allowances.

99. The collection of a $2,000 account within the 2 percent discount period will result in a
(a) debit to Sales Discounts for $40.
(b) debit to Accounts Receivable for $1,960.
(c) credit to Cash for $1,960.
(d) credit to Accounts Receivable for $1,960.

Copyright © 2014 John Wiley & Sons Canada, Ltd. Unauthorized copying, distribution, or transmission of this page is prohibited

Downloaded by Rnz Rnz (renzalconera@gmail.com)


lOMoARcPSD|4061633

5 - 18 Test Bank for Financial Accounting: Tools for Business Decision-Making, 6th Canadian Edition

100. Freight paid by the seller to a customer’s business is recorded as a


(a) credit to Sales.
(b) debit to Sales.
(c) debit to an operating expense.
(d) credit to Cost of Goods Sold.

101. If a customer agrees to keep defective merchandise because the seller is willing to reduce
the selling price, this transaction is known as a sales
(a) discount.
(b) return.
(c) contra asset.
(d) allowance.

102. When goods from a cash sale are returned, the effect on the seller’s accounts will be
(a) an increase in net sales.
(b) a decrease in gross sales.
(c) an increase in gross sales.
(d) a decrease in net sales.

103. Management may be alerted to a quality problem with their merchandise by a sudden
increase in which account?
(a) Sales
(b) Sales Returns and Allowances
(c) Sales Discounts
(d) Cost of Goods Sold

104. A Sales Returns and Allowances account is not debited if a customer


(a) returns defective merchandise.
(b) receives a credit for merchandise of inferior quality.
(c) pays within the discount period.
(d) returns goods that are not in accordance with specifications.

105. As an incentive for customers to pay their accounts promptly, a business may offer its
customers
(a) a sales discount.
(b) free delivery.
(c) a sales allowance.
(d) a sales return.

106. The credit terms offered by a company are 2/10, n/30, which means that
(a) the customer must pay the bill within 10 days.
(b) the customer can deduct a 2% discount if the bill is paid between 10 days and 30 days from
the invoice date.
(c) the customer can deduct a 2% discount if the bill is paid within 10 days of the invoice date.

Copyright © 2014 John Wiley & Sons Canada, Ltd. Unauthorized copying, distribution, or transmission of this page is prohibited

Downloaded by Rnz Rnz (renzalconera@gmail.com)


lOMoARcPSD|4061633

5 - 19 Merchandising Operations

(d) two sales returns can be made within 10 days of the invoice date and no returns thereafter.

107. A sales discount does not


(a) provide the purchaser with a cash saving.
(b) reduce the amount of cash received from a credit sale.
(c) increase a contra revenue account.
(d) increase an operating expense account.

108. Company A sells $500 of merchandise on account to Company B with credit terms of 2/10,
n/30. If Company B pays within the discount period, how much cash will Company A receive?
(a) $400
(b) $410
(c) $490
(d) $500

109. Chocolate Corporation sells merchandise on account for $3,000 to Marshmallow


Corporation with credit terms of 2/10, n/30. Marshmallow returns $600 of merchandise that was
damaged, along with a cheque to settle the account within the discount period. What is the
amount of the cheque?
(a) $2,952
(b) $2,940
(c) $2,400
(d) $2,352

110. Mountain Corp. sells merchandise on account for $2,000 to Cliff Corp., terms 2/10, n/30.
Cliff returns $800 worth of merchandise that was damaged, along with a cheque to settle the
account within the discount period. What entry does Mountain make upon receipt of the
cheque?
(a) Cash............................................................................................ 1,200
Accounts Receivable............................................................ 1,200
(b) Cash............................................................................................ 1,160
Sales Returns and Allowances.................................................... 784
Sales Discounts........................................................................... 32
Accounts Receivable............................................................ 2,000
(c) Cash............................................................................................ 1,176
Sales Returns and Allowances.................................................... 800
Sales Discounts........................................................................... 24
Accounts Receivable............................................................ 2,000
(d) Cash............................................................................................ 1,160
Sales Discounts........................................................................... 40
Sales Returns and Allowances.................................................... 800
Accounts Receivable............................................................ 2,000

111. The collection of a $1,000 account paid within the 2 percent discount period will result in a
(a) credit to Cash for $980.
(b) credit to Accounts Receivable for $1,000.

Copyright © 2014 John Wiley & Sons Canada, Ltd. Unauthorized copying, distribution, or transmission of this page is prohibited

Downloaded by Rnz Rnz (renzalconera@gmail.com)


lOMoARcPSD|4061633

5 - 20 Test Bank for Financial Accounting: Tools for Business Decision-Making, 6th Canadian Edition

(c) debit to Cash for $1,000.


(d) credit to Accounts Receivable for $980.

112. Which of the following would not be classified as a contra account?


(a) Sales
(b) Sales Returns and Allowances
(c) Accumulated Depreciation
(d) Sales Discounts

113. Which of the following accounts has a normal credit balance?


(a) Sales Returns and Allowances
(b) Sales Discounts
(c) Sales
(d) Cost of Goods Sold

114. The respective normal account balances of Sales, Sales Returns and Allowances, and
Sales Discounts are
(a) credit, credit, credit.
(b) debit, credit, debit.
(c) credit, debit, debit.
(d) credit, debit, credit.

115. Which one of the following would not appear on a single-step income statement?
(a) gross profit
(b) expenses
(c) sales revenues
(d) cost of goods sold

116. The form of the income statement that derives its name from the fact that the total of all
expenses is deducted from the total of all revenues is called a
(a) multiple-step income statement.
(b) revenue income statement.
(c) report-form income statement.
(d) single-step income statement.

117. Gross profit does not appear


(a) on a multiple-step income statement.
(b) on a single-step income statement.
(c) to be relevant in analyzing the operation of a merchandising company.
(d) on either a multiple-step or a single-step income statement.

118. Gross profit for a merchandising company equals the difference between net sales and
(a) operating expenses.
(b) cost of goods sold.

Copyright © 2014 John Wiley & Sons Canada, Ltd. Unauthorized copying, distribution, or transmission of this page is prohibited

Downloaded by Rnz Rnz (renzalconera@gmail.com)


lOMoARcPSD|4061633

5 - 21 Merchandising Operations

(c) profit.
(d) cost of goods sold plus operating expenses.

119. A loss from operations will result if operating expenses exceed


(a) cost of goods sold.
(b) selling expenses.
(c) cost of goods sold plus sales returns and allowances.
(d) gross profit.

120. What is the term applied to the excess of net sales over the cost of goods sold?
(a) gross sales
(b) profit from operations
(c) profit
(d) gross profit

121. Which of the following is not true about a multiple-step income statement?
(a) There is a section for operating expenses.
(b) There may be a section for non-operating activities.
(c) There may be a section for operating assets.
(d) There is a section for cost of goods sold.

122. An advantage of the single-step income statement over the multiple-step form is
(a) the amount of information it provides.
(b) its comprehensiveness.
(c) its simplicity.
(d) its use in calculating ratios.

123. Profit from operations appears on


(a) both a multiple-step and a single-step income statement.
(b) neither a multiple-step nor a single-step income statement.
(c) a single-step income statement only.
(d) a multiple-step income statement only.

124. Which statement is correct about expenses on the income statement?


(a) Classifying expenses by nature means that expenses are reported according to the activity
for which they are incurred.
(b) Examples of expenses classified by function are cost of goods sold and administrative
expenses.
(c) Expenses must be classified by their function.
(d) Expenses must be classified in decreasing order of magnitude.

125. Which statement is not correct about expenses on the income statement?
(a) Classifying expenses by function means that expenses are reported according to the activity
for which they are incurred.

Copyright © 2014 John Wiley & Sons Canada, Ltd. Unauthorized copying, distribution, or transmission of this page is prohibited

Downloaded by Rnz Rnz (renzalconera@gmail.com)


lOMoARcPSD|4061633

5 - 22 Test Bank for Financial Accounting: Tools for Business Decision-Making, 6th Canadian Edition

(b) Examples of expenses classified by nature are salaries and depreciation.


(c) Companies following ASPE do not have to list their expenses in any particular order.
(d) Expenses must be classified in decreasing order of magnitude.

126. A multiple-step income statement shows


(a) gross profit but not profit from operations.
(b) neither gross profit nor profit from operations.
(c) both gross profit and profit from operations.
(d) profit from operations but not gross profit.

127. Interest expense would be classified on a multiple-step income statement under the
heading
(a) Other expenses and losses.
(b) Other revenues and gains.
(c) Operating expenses.
(d) Cost of goods sold.

128. Profit from operations for a merchandising company is net sales less
(a) operating expenses.
(b) cost of goods sold.
(c) sales discounts and cost of goods sold.
(d) operating expenses and cost of goods sold.

129. The operating expenses section of a multiple-step income statement for a merchandising
company would not include
(a) freight out.
(b) utilities expense.
(c) cost of goods sold.
(d) loss on sale of equipment.

130. Which one of the following would appear on the income statement of both a merchandising
company and a service company?
(a) Gross profit
(b) Profit
(c) Sales revenues
(d) Cost of goods sold

131. Gross profit does not appear


(a) on a merchandising company’s multiple-step income statement.
(b) on a service company’s income statement.
(c) to be relevant in analyzing the operation of a merchandising company.
(d) on the income statement, if the periodic inventory system is used, because it cannot be
calculated.

Copyright © 2014 John Wiley & Sons Canada, Ltd. Unauthorized copying, distribution, or transmission of this page is prohibited

Downloaded by Rnz Rnz (renzalconera@gmail.com)


lOMoARcPSD|4061633

5 - 23 Merchandising Operations

Use the following information to answer questions 132–135.


Cost of goods sold.............................................................. $217,000
Income tax expense........................................................... 33,600
Operating expenses........................................................... 172,000
Sales.................................................................................. 550,000
Sales discounts.................................................................. 12,000
Sales returns and allowances............................................. 37,000

132. The amount of net sales on the income statement would be


(a) $501,000.
(b) $538,000.
(c) $513,000.
(d) $550,000.

133. Gross profit would be


(a) $112,000.
(b) $284,000.
(c) $378,000.
(d) $501,000.

134. The gross profit margin would be


(a) 56.7%.
(b) 34.3%.
(c) 43.3%.
(d) 39.5%.

135. The profit margin would be


(a) 18.5%.
(b) 15.6%.
(c) 60.6%.
(d) 34.3%.

136. The gross profit margin is calculated by dividing gross profit by


(a) sales.
(b) cost of goods sold.
(c) net sales.
(d) operating expenses.

137. A decline in a company’s gross profit could be caused by all of the following except
(a) selling products with a lower markup.
(b) clearance of discontinued inventory.
(c) paying lower prices to its suppliers.
(d) increased competition resulting in lower selling prices.

Copyright © 2014 John Wiley & Sons Canada, Ltd. Unauthorized copying, distribution, or transmission of this page is prohibited

Downloaded by Rnz Rnz (renzalconera@gmail.com)


lOMoARcPSD|4061633

5 - 24 Test Bank for Financial Accounting: Tools for Business Decision-Making, 6th Canadian Edition

138. If a company has net sales of $500,000 and cost of goods sold of $350,000, the gross
profit margin is
(a) 15%.
(b) 30%.
(c) 70%.
(d) 100%.

139. A company shows the following balances:


Cost of goods sold.............................................................. $ 900,000
Sales.................................................................................. 2,000,000
Sales discounts.................................................................. 25,000
Sales returns and allowances............................................. 225,000
What is the gross profit margin?
(a) 42.5%
(b) 48.6%
(c) 49.3%
(d) 55.0%

140. Profit margin is calculated by dividing


(a) profit by gross profit.
(b) profit by sales.
(c) profit by net sales.
(d) sales by profit.

141. Profit margin is a measure of


(a) liquidity.
(b) profitability.
(c) solvency.
(d) comparability.

142. Profit margin is calculated by dividing profit by


(a) sales.
(b) sales revenues.
(c) net sales.
(d) gross sales.

Use the following financial information to answer questions 143–144.


Operating expenses........................................................... $ 25,000
Sales returns and allowances............................................. 3,000
Sales.................................................................................. 110,000
Cost of goods sold.............................................................. 55,000
Income tax expense........................................................... 5,000

143. What is the gross profit margin?


(a) 20.6%

Copyright © 2014 John Wiley & Sons Canada, Ltd. Unauthorized copying, distribution, or transmission of this page is prohibited

Downloaded by Rnz Rnz (renzalconera@gmail.com)


lOMoARcPSD|4061633

5 - 25 Merchandising Operations

(b) 22.7%
(c) 48.6%
(d) 50.0%

144. What is the profit margin?


(a) 20.6%
(b) 22.7%
(c) 48.6%
(d) 50.0%

*145. Which of the following is not true for a company using a periodic inventory system?
(a) Cost of goods sold is calculated for each sale.
(b) Cost of goods sold is calculated at the end of the accounting period.
(c) A physical inventory count is performed at the end of the accounting period.
(d) Cost of goods available for sale is calculated at the end of the accounting period.

*146. Detailed records of goods held for resale are not maintained under a
(a) perpetual inventory system.
(b) periodic inventory system.
(c) double entry accounting system.
(d) single entry accounting system.

*147. Purchases less purchase returns and allowances less purchase discounts is called
(a) cost of goods purchased.
(b) net purchases.
(c) cost of goods sold.
(d) net inventory.

*148. Under a periodic inventory system, purchase of merchandise is debited to the


(a) Merchandise Inventory account.
(b) Cost of Goods Sold account.
(c) Purchases account.
(d) Accounts Payable account.

*149. Which of the following accounts has a normal credit balance?


(a) Purchases
(b) Sales Returns and Allowances
(c) Freight In
(d) Purchase Discounts

*150. The respective normal balances of Purchases, Purchase Discounts, and Freight In are
(a) credit, credit, debit.
(b) debit, credit, credit.
(c) debit, credit, debit.

Copyright © 2014 John Wiley & Sons Canada, Ltd. Unauthorized copying, distribution, or transmission of this page is prohibited

Downloaded by Rnz Rnz (renzalconera@gmail.com)


lOMoARcPSD|4061633

5 - 26 Test Bank for Financial Accounting: Tools for Business Decision-Making, 6th Canadian Edition

(d) debit, debit, debit.

*151. The Freight In account


(a) increases the cost of merchandise purchased.
(b) is a contra account to the Purchases account.
(c) is a permanent account.
(d) has a normal credit balance.

*152. Net purchases plus freight in is called


(a) cost of goods sold.
(b) cost of goods available for sale.
(c) cost of goods purchased.
(d) total goods available for sale.

*153. Beginning inventory plus the cost of goods purchased equals


(a) cost of goods sold.
(b) cost of goods available for sale.
(c) net purchases.
(d) total goods purchased.

*154. On the income statement, purchases less purchase discounts and purchase returns and
allowances, plus freight in equals
(a) cost of goods purchased.
(b) cost of goods available for sale.
(c) net purchases.
(d) gross profit.

*155. Benz Inc. shows the following account balances for last month:
Freight In............................................................................ $ 1,875
Freight Out......................................................................... 2,500
Purchases.......................................................................... 28,000
Purchase Discounts........................................................... 2,500
Sales Returns and Allowances........................................... 4,000
The cost of goods purchased for last month is
(a) $25,875.
(b) $27,375.
(c) $29,875.
(d) $30,500.

*156. Stylish Shoe Store reported beginning merchandise inventory of $15,000. During the
period, purchases were $70,000; purchase returns, $2,000; and freight in $5,000. A physical
count of inventory at the end of the period revealed that $10,000 was still on hand. The cost of
goods available for sale was
(a) $78,000.
(b) $82,000.

Copyright © 2014 John Wiley & Sons Canada, Ltd. Unauthorized copying, distribution, or transmission of this page is prohibited

Downloaded by Rnz Rnz (renzalconera@gmail.com)


lOMoARcPSD|4061633

5 - 27 Merchandising Operations

(c) $88,000.
(d) $92,000.

*157. Cost of goods sold is calculated from the following equation:


(a) beginning inventory – cost of goods purchased + ending inventory.
(b) sales – cost of goods purchased + beginning inventory – ending inventory.
(c) sales + gross profit – ending inventory + beginning inventory.
(d) beginning inventory + cost of goods purchased – ending inventory.

Use the following information to answer questions *158–*160.


For last month, the following data were taken from the ledger of Drillbit Inc.:
Beginning Inventory............................................................ $ 21,500
Ending Inventory................................................................ 16,200
Freight In............................................................................ 1,150
Purchases.......................................................................... 112,000
Purchase Discounts........................................................... 750
Purchase Returns and Allowances..................................... 1,900

*158. What was the cost of goods purchased?


(a) $110,100
(b) $109,350
(c) $110,500
(d) $108,200

*159. What was the cost of goods sold?


(a) $117,300
(b) $115,800
(c) $118,800
(d) $106,700

*160. What was the cost of goods available for sale?


(a) $132,000
(b) $133,500
(c) $134,650
(d) $117,300

*161. On the income statement, the beginning merchandise inventory is added to the cost of
goods purchased to determine the
(a) cost of goods sold.
(b) cost of goods available for sale.
(c) profit from operations.
(d) gross profit.

Copyright © 2014 John Wiley & Sons Canada, Ltd. Unauthorized copying, distribution, or transmission of this page is prohibited

Downloaded by Rnz Rnz (renzalconera@gmail.com)


lOMoARcPSD|4061633

5 - 28 Test Bank for Financial Accounting: Tools for Business Decision-Making, 6th Canadian Edition

ANSWERS TO MULTIPLE CHOICE QUESTIONS

Item Ans. Item Ans. Item Ans. Item Ans. Item Ans. Item Ans. Item Ans.
58. c 73. d 88. d 103. b 118. b 133. b *148. c
59. b 74. c 89. c 104. c 119. d 134. a *149. d
60. c 75. c 90. c 105. a 120. d 135. b *150. c
61. b 76. d 91. c 106. c 121. c 136. c *151. a
62. d 77. c 92. a 107. d 122. c 137. c *152. c
63. a 78. b 93. d 108. c 123. d 138. b *153. b
64. c 79. b 94. b 109. d 124. b 139. b *154. a
65. a 80. a 95. d 110. c 125. d 140. c *155. b
66. b 81. b 96. b 111. b 126. c 141. b *156. c
67. a 82. c 97. d 112. a 127. a 142. c *157. d
68. b 83. d 98. c 113. c 128. d 143. c *158. c
69. d 84. a 99. a 114. c 129. c 144. a *159. b
70. a 85. a 100. c 115. a 130. b *145. a *160. a
71. b 86. b 101. d 116. d 131. b *146. b *161. b
72. d 87. b 102. d 117. b 132. a *147. b

Copyright © 2014 John Wiley & Sons Canada, Ltd. Unauthorized copying, distribution, or transmission of this page is prohibited

Downloaded by Rnz Rnz (renzalconera@gmail.com)


lOMoARcPSD|4061633

5 - 29 Merchandising Operations

EXERCISES

Ex. 162
Sherla Holmes is a new accountant with Moriarty Corporation. Moriarty purchased merchandise
on account for $5,000. The credit terms are 2/10, n/30. Sherla has talked with the company's
banker and knows that she could earn 9% on any money invested in the company's savings
account.

Instructions
(a) Should Sherla pay the invoice within the discount period or should she keep the $5,000 in
the savings account and pay at the end of the credit period (i.e., after 30 days)? Support
your recommendation with a calculation showing which action would be best.
(b) If Sherla forgoes the discount, it may be viewed as paying an interest rate of 2% for the use
of $5,000 for 20 days. Calculate the annual rate of interest that this is equivalent to.

Solution 162 (10 min.)


(a) Discount of 2% on $5,000 $100.00
Interest received on $5,000 (for 20 days at 9%) $24.66 ($5,000  9%  20  365)
Savings by taking the discount $75.34
Recommendation: Sherla should pay the invoice within the discount period.

(b) The equivalent annual interest rate is:


2%  365  20 = 36.5%.

Ex. 163
Jun 4 Willem Corporation purchased $4,000 worth of merchandise, terms 2/10, n/30 from
Cate Corporation. The cost of the merchandise to Cate was $2,600.
10 Willem returned $700 worth of goods to Cate for full credit. The goods had a cost of
$450 to Cate and were placed back into inventory.
26 Willem paid the account.

Instructions
Prepare the journal entries to record these transactions in (a) Willem’s records and (b) Cate’s
records. Both companies use the perpetual inventory system.

Solution 163 (15–20 min.)


(a) Willem’s records
Jun 4 Merchandise Inventory....................................................... 4,000
Accounts Payable........................................................ 4,000

10 Accounts Payable............................................................... 700


Merchandise Inventory................................................ 700

26 Accounts Payable ($4,000 – $700)..................................... 3,300


Cash............................................................................ 3,300

(b) Cate’s records


Jun 4 Accounts Receivable.......................................................... 4,000

Copyright © 2014 John Wiley & Sons Canada, Ltd. Unauthorized copying, distribution, or transmission of this page is prohibited

Downloaded by Rnz Rnz (renzalconera@gmail.com)


lOMoARcPSD|4061633

5 - 30 Test Bank for Financial Accounting: Tools for Business Decision-Making, 6th Canadian Edition

Sales........................................................................... 4,000

4 Cost of Goods Sold............................................................ 2,600


Merchandise Inventory................................................ 2,600

10 Sales Returns and Allowance............................................. 700


Accounts Receivable................................................... 700

10 Merchandise Inventory....................................................... 450


Cost of Goods Sold..................................................... 450

26 Cash................................................................................... 3,300
Accounts Receivable ($4,000 – $700)......................... 3,300

Ex. 164
On July 1, Ricker Cycle Shop had an inventory of 20 bicycles at a cost of $250 each. Ricker
uses a perpetual inventory system. During the month of July, the following transactions
occurred:
Jul 4 Purchased 25 bicycles at a cost of $250 each from the Joncas Bicycle Corporation,
terms 2/10, n/30.
5 Paid freight of $125 on the July 4 purchase.
6 Sold 10 bicycles from the July 1 inventory to Team Canada for $350 each, terms
2/10, n/30.
7 Received a credit from Joncas Bicycle for the return of 2 defective bicycles.
8 Sold two bicycles from the July 1 inventory for $700 cash.
13 Issued a credit memo to Team Canada for the return of a defective bicycle.
14 Paid Joncas Bicycle in full.
15 Received payment from Team Canada.

Instructions
Record the July transactions for Ricker Cycle Shop.

Solution 164 (20 min.)


Jul 4 Merchandise Inventory ($250 x 25).................................... 6,250
Accounts Payable........................................................ 6,250

5 Merchandise Inventory....................................................... 125


Cash............................................................................ 125

6 Accounts Receivable ($350 x 10)....................................... 3.500


Sales........................................................................... 3.500
Cost of Goods Sold ($250 x 10)......................................... 2,500
Merchandise Inventory................................................ 2,500

7 Accounts Payable............................................................... 500


Merchandise Inventory................................................ 500

8 Cash................................................................................... 700

Copyright © 2014 John Wiley & Sons Canada, Ltd. Unauthorized copying, distribution, or transmission of this page is prohibited

Downloaded by Rnz Rnz (renzalconera@gmail.com)


lOMoARcPSD|4061633

5 - 31 Merchandising Operations

Sales........................................................................... 700
Cost of Goods Sold ($250 x 2)........................................... 500
Merchandise Inventory................................................ 500

13 Sales Returns and Allowances........................................... 350


Accounts Receivable................................................... 350

14 Accounts Payable ($6,250 – $500)..................................... 5,750


Cash ($5,750  98%).................................................. 5,635
Merchandise Inventory ($5,750  2%)........................ 115

15 Cash ($3,150 x 98%).......................................................... 3,087


Sales Discounts ($3,150 x 2%).......................................... 63
Accounts Receivable ($3,500 – $350)......................... 3,150

Ex. 165
On September 1, Wilderness Inc. had an inventory of 18 backpacks at a cost of $30 each. The
company uses a perpetual inventory system. During September, the following transactions
occurred:
Sep 4 Purchased 35 backpacks at $30 each from Back Packs Unlimited, terms 3/10, n/30.
6 Received credit of $150 for the return of 5 backpacks purchased on Sept. 4 that
were defective.
9 Sold 20 backpacks for $50 each to University Supply, terms 2/10, n/30.
14 Paid Back Packs Unlimited in full.
18 Received payment from University Supply.

Instructions
Record the September transactions for Wilderness Inc.

Solution 165 (15–20 min.)


Sep 4 Merchandise Inventory ($30 x 35)...................................... 1,050
Accounts Payable........................................................ 1,050

6 Accounts Payable............................................................... 150


Merchandise Inventory................................................ 150

9 Accounts Receivable ($50 x 20)......................................... 1,000


Sales........................................................................... 1,000

Cost of Goods Sold ($30 x 20)........................................... 600


Merchandise Inventory................................................ 600

14 Accounts Payable ($1,050 – $150)..................................... 900


Cash ($900  97%)..................................................... 873
Merchandise Inventory ($900  3%)........................... 27

18 Cash ($1,000 x 98%).......................................................... 980


Sales Discounts ($1,000 x 2%).......................................... 20
Accounts Receivable................................................... 1,000

Copyright © 2014 John Wiley & Sons Canada, Ltd. Unauthorized copying, distribution, or transmission of this page is prohibited

Downloaded by Rnz Rnz (renzalconera@gmail.com)


lOMoARcPSD|4061633

5 - 32 Test Bank for Financial Accounting: Tools for Business Decision-Making, 6th Canadian Edition

Ex. 166
Gia’s Gymnastics Gear uses a perpetual inventory system. The following transactions occurred
in July:
Jul 6 Purchased $1,800 of merchandise on credit, terms 1/10, n/30.
8 Because some of the items purchased on July 6 had a small defect, Gia’s
Gymnastics Gear received a purchase allowance of $175.
9 Paid freight charges of $75 on the items purchased July 6.
19 Sold merchandise on credit for $1,800, terms 2/10, n/30. The merchandise had a
cost of $900.
22 Of the merchandise sold on July 19, $200 of it was returned. The items had cost
Gia’s$100 and were returned to inventory.
28 Received payment from the customer of July 19.
31 Paid for the merchandise purchased on July 6.

Instructions
Record the July transactions for Gia’s Gymnastics Gear.

Solution 166 (15–20 min.)


Jul 6 Merchandise Inventory....................................................... 1,800
Accounts Payable........................................................ 1,800

8 Accounts Payable............................................................... 175


Merchandise Inventory................................................ 175

9 Merchandise Inventory....................................................... 75
Cash............................................................................ 75

19 Accounts Receivable.......................................................... 1,800


Sales........................................................................... 1,800

Cost of Goods Sold............................................................ 900


Merchandise Inventory................................................ 900

22 Sales Returns and Allowances........................................... 200


Accounts Receivable................................................... 200

Merchandise Inventory....................................................... 100


Cost of Goods Sold..................................................... 100

28 Cash ($1,600 x 98%).......................................................... 1,568


Sales Discount ($1,600 x 2%)............................................ 32
Accounts Receivable ($1,800 – $200)......................... 1,600

31 Accounts Payable ($1,800 – $175)..................................... 1,625


Cash............................................................................ 1,625

Ex. 167

Copyright © 2014 John Wiley & Sons Canada, Ltd. Unauthorized copying, distribution, or transmission of this page is prohibited

Downloaded by Rnz Rnz (renzalconera@gmail.com)


lOMoARcPSD|4061633

5 - 33 Merchandising Operations

(a) Sean Corporation purchased merchandise on account from Kingston Supplies for $68,000,
with terms of 2/10, n/30. During the discount period, Sean returned some merchandise and
paid $56,840 as payment in full. Sean uses a perpetual inventory system. Prepare the
journal entries that Sean made to record the
1. purchase of merchandise.
2. return of merchandise.
3. payment on account.

(b) Willow Corporation sold merchandise to Jada Corporation on account for $84,000 with
credit terms of ?/10, n/30. The cost of the merchandise sold was $42,000. During the
discount period, Jada returned $14,000 worth of merchandise and paid its account in full
(minus the return and the discount) by paying $67,200 in cash. The returned goods were
returned to inventory. Both companies use a perpetual inventory system. Prepare the
journal entries that Willow Corporation made to record the
1. sale of merchandise.
2. return of merchandise.
3. collection on account.

Solution 167 (15–20 min.)


(a) To calculate the amount due after returns but before the discount, divide $56,840 by 98%
(100% – 2%) = $56,840  98% = $58,000
Subtract $58,000 from $68,000 to determine that $10,000 of merchandise was returned.

1. Merchandise Inventory............................................................ 68,000


Accounts Payable............................................................ 68,000

2. Accounts Payable................................................................... 10,000


Merchandise Inventory..................................................... 10,000

3. Accounts Payable................................................................... 58,000


Merchandise Inventory (58,000 x 2%).............................. 1,160
Cash................................................................................ 56,840

(b) Jada returns $14,000 of merchandise and thus owes $70,000 to Willow.
$67,200  $70,000 = 96%; 100% – 96% = 4%
The missing discount percentage is 4%. $70,000  4% = $2,800 sales discount
$70,000 – $2,800 = $67,200 cash received on account

1. Accounts Receivable.............................................................. 84,000


Sales................................................................................ 84,000

Cost of Goods Sold................................................................. 42,000


Merchandise Inventory..................................................... 42,000

2. Sales Returns and Allowances............................................... 14,000


Accounts Receivable........................................................ 14,000

Merchandise Inventory................................................................. 7,000


Cost of Goods Sold................................................................. 7,000

Copyright © 2014 John Wiley & Sons Canada, Ltd. Unauthorized copying, distribution, or transmission of this page is prohibited

Downloaded by Rnz Rnz (renzalconera@gmail.com)


lOMoARcPSD|4061633

5 - 34 Test Bank for Financial Accounting: Tools for Business Decision-Making, 6th Canadian Edition

3. Cash....................................................................................... 67,200
Sales Discounts...................................................................... 2,800
Accounts Receivable........................................................ 70,000

Ex. 168
Presented below are selected transactions for Scotian Corporation during July.
Jul 1 Sold merchandise to Brunswick Inc. for $800, terms 3/10, n/30. The merchandise
sold cost $400.
2 Purchased merchandise from Founders Corporation for $4,500, terms 4/10, n/30.
3 Paid freight charges of $100 on items purchased on July 2.
4 Purchased merchandise from Edward Company Ltd. for $5,000, n/30.
10 Received payment from Brunswick Inc. for purchase of July 1.
11 Paid Founders Corporation for July 2 purchase.

Instructions
(a) Record the above transactions for Scotian Corporation, assuming a perpetual inventory
system is used. The cost of goods sold on July 1 was determined to be $400.
(b) Record the above transactions for Scotian Corporation, assuming a periodic inventory
system is used.

Solution 168 (25 min.)


(a) Perpetual
Jul 1 Accounts Receivable.......................................................... 800
Sales......................................................................... 800

Cost of Goods Sold............................................................ 400


Merchandise Inventory.............................................. 400

2 Merchandise Inventory....................................................... 4,500


Accounts Payable .................................................... 4,500

3 Merchandise Inventory....................................................... 100


Cash.......................................................................... 100

4 Merchandise Inventory....................................................... 5,000


Accounts Payable .................................................... 5,000

10 Cash ($800 x 97%)............................................................. 776


Sales Discounts ($800 x 3%)............................................. 24
Accounts Receivable................................................. 800

11 Accounts Payable............................................................... 4,500


Merchandise Inventory ($4,500 x 4%)....................... 180
Cash ($4,500 x 96%)................................................ 4,320

(b) Periodic
Jul 1 Accounts Receivable.......................................................... 800
Sales ........................................................................ 800

Copyright © 2014 John Wiley & Sons Canada, Ltd. Unauthorized copying, distribution, or transmission of this page is prohibited

Downloaded by Rnz Rnz (renzalconera@gmail.com)


lOMoARcPSD|4061633

5 - 35 Merchandising Operations

2 Purchases.......................................................................... 4,500
Accounts Payable .................................................... 4,500

3 Freight-in............................................................................ 100
Cash....................................................................... 100

4 Purchases.......................................................................... 5,000
Accounts Payable .................................................... 5,000

10 Cash ($800 x 97%)............................................................. 776


Sales Discounts ($800 x 3%)............................................. 24
Accounts Receivable................................................. 800

11 Accounts Payable............................................................... 4,500


Purchase Discounts ($4,500 x 4%)........................... 180
Cash ($4,500 x 96%)................................................ 4,320

Ex. 169
The following table summarizes the sales for the month of July for Perfect Platters Wholesalers
Inc. The table includes the terms, sales returns and when payment was collected for each sale.

Date Sale Amount Terms Returns Date Collected


April 3 $ 900 2/10, n/30 $ 50 April 9
April 5 1,300 3/10, n/30 200 April 21
April 11 450 1/10, n/30 0 April 13
April 18 2,300 4/10, n/60 520 April 25
April 22 1,600 2/10, n/30 750 May 5

Instructions
Calculate the cash received from each sale. Show your calculations.

Solution 169 (10 min.)


Apr 3 $ 833 ($900 – $50 = $850; $850 x 2% = $17; $850 – $17 = $833)

Apr 5 $ 1,100 ($1,300 – $200 = $1,100; discount not taken)

Apr 11 $ 445.50 ($450 x 1% = $4.50; $450 – $4.50 = $445.50)

Apr 18 $ 1,708.80 ($2,300 – $520 = $1,780; $1,780 x 4% = $71.20; $1,780 – $71.20 =


$1,708.80)

Apr 22 $ 850 ($1,600 – $750 = $850; discount not taken)

Ex. 170
Storm Inc. completed the following transactions in October:

Credit Sales Sales Returns Date of


Date Amount Terms Date Amount Collection

Copyright © 2014 John Wiley & Sons Canada, Ltd. Unauthorized copying, distribution, or transmission of this page is prohibited

Downloaded by Rnz Rnz (renzalconera@gmail.com)


lOMoARcPSD|4061633

5 - 36 Test Bank for Financial Accounting: Tools for Business Decision-Making, 6th Canadian Edition

Oct 3 $ 800 2/10, n/30 Oct 8


11 1,200 3/10, n/30 Oct 14 $ 500 16
17 7,000 1/10, n/30 20 1,200 29
21 1,700 2/10, n/60 23 400 27
23 2,500 2/10, n/30 27 500 28

Storm uses a perpetual inventory system.

Instructions
(a) Calculate the cash received for each collection. Show your calculations.
(b) Prepare the journal entry for the
(1) Oct. 17 sale. The merchandise sold had a cost of $3,500.
(2) Oct. 23 sales return. The merchandise returned had a cost of $200 and was returned to
inventory.
(3) Oct. 28 collection.

Solution 170 (20 min.)


(a)
Oct 8 $784 [Sales $800 – Sales discount $16 ($800  2%)]

16 $679 [Sales $1,200 – Sales return $500 = $700;


$700 – Sales discount $21 ($700  3%)]

29 $5,800 [Sales $7,000 – Sales return $1,200 = $5,800;


(discount not taken)]

27 $1,274 [Sales $1,700 – Sales return $400 = $1,300;


$1,300 – Sales discount $26 ($1,300  2%)]

28 $1,960 [Sales $2,500 – Sales return $500 = $2,000;


$2,000 – Sales discount $40 ($2,000  2%)]

(b)
(1) Oct 17 Accounts Receivable..................................................... 7,000
Sales....................................................................... 7,000
Cost of Goods Sold........................................................ 3,500
Merchandise Inventory............................................ 3,500

(2) 23 Sales Returns and Allowances...................................... 400


Accounts Receivable.............................................. 400
Merchandise Inventory.................................................. 200
Cost of Goods Sold................................................. 200

(3) 28 Cash.............................................................................. 1,960


Sales Discounts............................................................. 40
Accounts Receivable.............................................. 2,000

Ex. 171
Financial information is presented here for two companies. Complete the missing amounts.

Copyright © 2014 John Wiley & Sons Canada, Ltd. Unauthorized copying, distribution, or transmission of this page is prohibited

Downloaded by Rnz Rnz (renzalconera@gmail.com)


lOMoARcPSD|4061633

5 - 37 Merchandising Operations

Empty Corporation Full Corporation


Cost of goods sold $26,000 $ ?
Gross profit ? 38,000
Income tax expense 6,500 9,000
Net sales 47,000 62,000
Operating expenses 8,000 ?
Profit ? 9,000
Profit before income tax 13,000 18,000
Sales 50,000 ?
Sales returns ? 5,000

Solution 171 (15 min.)


Empty Corporation
Sales returns = $3,000($50,000 – $47,000 = $3,000)
Gross profit = $21,000($47,000 – $26,000 = $21,000)
Profit = $6,500($21,000 – $8,000 – $6,500 = $6,500)

Full Corporation
Sales = $67,000($62,000 + $5,000 = $67,000)
Cost of goods sold = $24,000($62,000 – $38,000 = $24,000)
Operating expenses = $20,000($38,000 – $18,000 = $20,000)

Ex. 172
State the missing items identified by ?.
(a) Gross profit – Operating expenses = ?
(b) Sales – (? + ?) = Net sales
(c) Profit from operations + ? – ? = Profit before income tax
(d) Net sales – Cost of goods sold = ?
(e) Cost of goods sold + Gross profit = ?

Solution 172 (5 min.)


(a) Profit from operations

(b) Sales discounts, Sales returns and allowances

(c) Other revenues and gains, Other expenses and losses

(d) Gross profit

(e) Net sales

Ex. 173
The following information was taken from the adjusted trial balance of Lucifer Lighting Inc. at
December 31, 2015. All accounts have normal balances.
Accounts payable............................................... $ 52,000
Accounts receivable........................................... 18,700

Copyright © 2014 John Wiley & Sons Canada, Ltd. Unauthorized copying, distribution, or transmission of this page is prohibited

Downloaded by Rnz Rnz (renzalconera@gmail.com)


lOMoARcPSD|4061633

5 - 38 Test Bank for Financial Accounting: Tools for Business Decision-Making, 6th Canadian Edition

Accumulated depreciation—Building.................. 44,900


Advertising expense........................................... 38,500
Building.............................................................. 600,000
Cash................................................................... 85,000
Common shares................................................. 417,500
Cost of goods sold.............................................. 410,500
Depreciation expense......................................... 12,000
Freight out.......................................................... 22,000
Interest expense................................................. 5,700
Interest revenue................................................. 2,000
Rental revenue................................................... 6,000
Retained earnings, Jan 1................................... 154,800
Salaries expense................................................ 279,500
Salaries payable................................................. 5,200
Sales ................................................................. 798,500
Sales discounts ................................................. 8,200
Sales returns and allowances ............................ 29,000
Utilities expense . 9,200

Instructions
Use the above information to prepare a multiple-step income statement for the year ended
December 31, 2015.

Solution 173
LUCIFER LIGHTING INC.
Income Statement
Year Ended December 31, 2015
___________________________________________________________________________
Sales.................................................................................................. $798,500
Less: Sales returns and allowances................................................. $ 29,000
Sales discounts....................................................................... 8,200 37,200
Net sales............................................................................................ 761,300
Cost of goods sold.............................................................................. 410,500
Gross profit......................................................................................... 350,800
Operating expenses
Salaries expense......................................................................... $279,500
Advertising expense.................................................................... 38,500
Freight out................................................................................... 22,000
Depreciation expense.................................................................. 12,000
Utilities expense.......................................................................... 9,200
Total operating expenses..................................................... 361,200
Loss from operations.......................................................................... (10,400)
Other revenues and gains
Interest revenue.......................................................................... $ 2,000
Rental revenue............................................................................ 6,000
Other expenses and losses
Interest expense.......................................................................... 5,700 2,300
Loss................................................................................................... $ (8,100)

Ex. 174

Copyright © 2014 John Wiley & Sons Canada, Ltd. Unauthorized copying, distribution, or transmission of this page is prohibited

Downloaded by Rnz Rnz (renzalconera@gmail.com)


lOMoARcPSD|4061633

5 - 39 Merchandising Operations

Financial information is presented here for two companies:


Company A Company B
Cost of goods sold........................... $385,000 $ ?
Gross profit...................................... 395,000 438,000
Income tax expense......................... 38,000 ?
Net sales.......................................... 780,000 923,000
Operating expenses......................... ? 190,000
Profit................................................ ? 198,400
Profit before income tax................... 190,000 248,000
Sales................................................ ? 950,000
Sales discounts................................ 6,000 ?
Sales returns and allowances.......... 14,000 18,000

Instructions
(a) Calculate the missing amounts for each company
(b) For each company, calculate the gross profit margin and the profit margin.
(c) Which company is more profitable?

Solution 174 (20 min.)


(a) Company A
Sales $800,000 ($780,000 + $6,000 + $14,000)
Operating expenses $205,000 ($395,000 – $190,000)
Profit $152,000 ($190,000 – $38,000)

Company B
Sales discounts $9,000 ($950,000 – $18,000 – $923,000)
Cost of goods sold $485,000 ($923,000 – $438,000)
Income tax expense $49,600 ($248,000 – $198,400)

(b) Company A
Gross profit margin = 50.6% ($395,000 ÷ $780,000)
Profit margin = 19.5% ($152,000 ÷ $780,000)

Company B
Gross profit margin = 47.5% ($438,000 ÷ $923,000)
Profit margin = 21.5% ($198,400 ÷ $923,000)

(c) Although Company A has a higher gross profit margin, Company B is more profitable.

Ex. 175
The following information is available for Shawson Ltd. for calendar 2015:
Cost of goods sold....................................................... 595,000
Income tax expense.................................................... 4,500
Interest expense.......................................................... 15,000
Interest revenue.......................................................... 19,000
Operating expenses.................................................... 97,000
Sales........................................................................... $725,000
Sales returns and allowances...................................... 22,000

Copyright © 2014 John Wiley & Sons Canada, Ltd. Unauthorized copying, distribution, or transmission of this page is prohibited

Downloaded by Rnz Rnz (renzalconera@gmail.com)


lOMoARcPSD|4061633

5 - 40 Test Bank for Financial Accounting: Tools for Business Decision-Making, 6th Canadian Edition

Instructions
(a) Use the above information to prepare a multiple-step income statement for the year ended
December 31, 2015.
(b) Calculate the gross profit margin and the profit margin for 2015.

Solution 175 (20 min.)


(a) SHAWSON LTD.
Income Statement
Year Ended December 31, 2015
___________________________________________________________________________
Sales revenues
Sales.............................................................................................. $725,000
Less: Sales returns and allowances.............................................. 22,000
Net sales........................................................................................ 703,000
Cost of goods sold......................................................................... 595,000
Gross profit.................................................................................... 108,000
Operating expenses....................................................................... 97,000
Profit from operations.................................................................... 11,000
Other revenues and gains
Interest revenue...................................................................... $19,000
Other expenses and losses
Interest expense..................................................................... 15,000 (4,000)
Profit before income tax................................................................. 15,000
Income tax expense....................................................................... 4,500
Profit.............................................................................................. $ 10,500

(b) Gross profit margin: $108,000 ÷ $703,000 = 15.4%


Profit margin: $10,500  $703,000 = 1.5%

Ex. 176
The adjusted trial balance of Sandhu Corporation at December 31, 2015 included the following
selected accounts:
Debit Credit
Advertising expense........................................... $ 15,000
Cost of goods sold.............................................. 347,000
Depreciation expense......................................... 3,296
Freight out.......................................................... 2,000
Income tax expense........................................... 32,000
Interest expense................................................. 19,000
Interest revenue................................................. $ 15,000
Sales.................................................................. 575,000
Sales discounts.................................................. 10,500
Sales returns and allowances............................. 55,000
Store salaries expense....................................... 45,000
Utilities expense................................................. 18,000

Instructions

Copyright © 2014 John Wiley & Sons Canada, Ltd. Unauthorized copying, distribution, or transmission of this page is prohibited

Downloaded by Rnz Rnz (renzalconera@gmail.com)


lOMoARcPSD|4061633

5 - 41 Merchandising Operations

(a) Use the above information to prepare a multiple-step income statement for the year ended
December 31, 2015.
(b) Calculate the gross profit margin and the profit margin for 2015.

Solution 176 (25 min.)


(a)
SANDHU CORPORATION
Income Statement
Year Ended December 31, 2015
___________________________________________________________________________
Sales.................................................................................................... $575,000
Less: Sales returns and allowances................................................... $55,000
Sales discounts......................................................................... 10,500 65,500
Net sales.............................................................................................. 509,500
Cost of goods sold................................................................................ 347,000
Gross profit........................................................................................... 162,500
Operating expenses
Store salaries expense.................................................................. $45,000
Utilities expense............................................................................. 18,000
Advertising expense...................................................................... 15,000
Depreciation expense.................................................................... 3,296
Freight out..................................................................................... 2,000
Total operating expenses........................................................ 83,296
Profit from operations........................................................................... 79,204
Other revenues and gains
Interest revenue............................................................................. $15,000
Other expenses and losses
Interest expense............................................................................ 19,000 4,000
Profit before income tax........................................................................ 75,204
Income tax expense............................................................................. 32,000
Profit..................................................................................................... $ 43,204

(b) Gross profit margin = $162,500 ÷ $509,500 = 31.9%


Profit margin = $43,204 ÷ $509,500 = 8.5%

Ex. 177
The adjusted trial balance of Jayco Corporation at December 31, 2015 included the following
selected accounts:
Debit Credit
Advertising expense........................................... $ 45,000
Cost of goods sold.............................................. 592,000
Depreciation expense......................................... 4,200
Freight out.......................................................... 11,200
Income tax expense........................................... 74,280
Interest expense................................................. 12,500
Interest revenue................................................. $ 15,000
Salaries expense................................................ 248,000
Sales.................................................................. 1,200,000

Copyright © 2014 John Wiley & Sons Canada, Ltd. Unauthorized copying, distribution, or transmission of this page is prohibited

Downloaded by Rnz Rnz (renzalconera@gmail.com)


lOMoARcPSD|4061633

5 - 42 Test Bank for Financial Accounting: Tools for Business Decision-Making, 6th Canadian Edition

Sales discounts.................................................. 8,000


Sales returns and allowances............................. 34,000
Utilities expense................................................. 12,500

Instructions
(a) Use the above information to prepare a multiple-step income statement for the year ended
December 31, 2015.
(b) Calculate the gross profit margin and the profit margin for 2015.

Solution 177 (25 min.)


(a)
JAYCO CORPORATION
Income Statement
Year Ended December 31, 2015
___________________________________________________________________________
Sales.................................................................................................. $1,200,000
Less: Sales returns and allowances................................................. $ 34,000
Sales discounts....................................................................... 8,000 42,000
Net sales............................................................................................ 1,158,000
Cost of goods sold.............................................................................. 592,000
Gross profit......................................................................................... 566,000
Operating expenses
Salaries expense......................................................................... $248,000
Advertising expense.................................................................... 45,000
Utilities expense.......................................................................... 12,500
Freight out................................................................................... 11,200
Depreciation expense.................................................................. 4,200
Total operating expenses..................................................... 320,900
Profit from operations ........................................................................ 245,100
Other revenues and gains
Interest revenue.......................................................................... $ 15,000
Other expenses and losses
Interest expense.......................................................................... 12,500 2, 500
Profit before income tax..................................................................... 247,600
Income tax expense........................................................................... 74,280
Profit................................................................................................... $ 173,320

(b) Gross profit margin = $566,000 ÷ $1,158,000 = 48.9%


Profit margin = $173,320 ÷ $1,158,000 = 15.0%

Ex. 178
The following information is available from recent financial statements of Competitor A and
Competitor B:
(Amounts in millions)
Competitor A Competitor B
Cost of goods sold.................................... $21,761 $27,257
Income tax expense.................................. 361 766
Net sales................................................... 29,656 36,704
Operating expenses.................................. 7,962 10,435

Copyright © 2014 John Wiley & Sons Canada, Ltd. Unauthorized copying, distribution, or transmission of this page is prohibited

Downloaded by Rnz Rnz (renzalconera@gmail.com)


lOMoARcPSD|4061633

5 - 43 Merchandising Operations

Profit.......................................................... 594 1,072


Profit before income tax............................ 955 1,838

Instructions
(a) Calculate the profit margin and gross profit margin for each company.
(b) What conclusions can be drawn from the ratios calculated in part (a) about the relative
profitability of the two companies?

Solution 178 (15 min.)


(a)
Competitor A Competitor B
Profit margin: $594 $1,072
———— = 2.0% ———— = 2.9%
$29,656 $36,704

Gross profit margin: $29,656 – $21,761 $36,704 – $27,257


————————– ————————–
$29,656 $36,704

$7,895 $9,447
———— = 26.6% ———— = 25.7%
$29,656 $36,704

(b) Competitor B’s profit margin was 45% higher [(2.9% – 2.0%) ÷ 2.0%] than Competitor A’s,
but Competitor A’s gross profit margin was 3.5% higher [(26.6% – 25.7%) ÷ 25.7%] than
Competitor B’s margin. It can be concluded that Competitor B was slightly more profitable
than Competitor A because its profit margin was higher.

Ex. 179
Summarized below are the transactions recorded by Rummy Ltd. for calendar 2015, using a
perpetual inventory system. Their Jan 1 opening balances were: accounts receivable $145,000,
inventory $45,000, and accounts payable $122,000.

Merchandise Inventory......................................................................... 400,000


Accounts Payable.......................................................................... 400,000
(Purchase of inventory)

Merchandise Inventory......................................................................... 10,000


Cash.............................................................................................. 10,000
(Payment of freight-in on inventory)

Accounts Payable................................................................................. 20,000


Merchandise Inventory.................................................................. 20,000
(Returned merchandise to supplier for credit)

Accounts Receivable............................................................................ 538,000


Cash..................................................................................................... 200,000
Sales.............................................................................................. 738,000
(Record sales for year)

Copyright © 2014 John Wiley & Sons Canada, Ltd. Unauthorized copying, distribution, or transmission of this page is prohibited

Downloaded by Rnz Rnz (renzalconera@gmail.com)


lOMoARcPSD|4061633

5 - 44 Test Bank for Financial Accounting: Tools for Business Decision-Making, 6th Canadian Edition

Cost of Goods Sold.............................................................................. 420,000


Merchandise Inventory.................................................................. 420,000
(Record COGS for year)

Sales Returns and Allowances............................................................. 28,000


Accounts Receivable..................................................................... 28,000
(Record goods returned from customers)

Merchandise Inventory......................................................................... 16,800


Cost of Goods Sold........................................................................ 16,800
(Record goods returned from customers)

Accounts Payable................................................................................. 400,000


Merchandise Inventory.................................................................. 6,000
Cash.............................................................................................. 394,000
(Record payments to suppliers, with a $6,000 purchase discount)

Cash..................................................................................................... 560,000
Sales Discounts.................................................................................... 4,000
Accounts Receivable..................................................................... 564,000
(Record receipts from customers)

Instructions
Prepare the 2015 income statement to the gross profit line only.
(a) As it would appear using the perpetual inventory system.
(b) As it would appear if a periodic inventory system had been used.
(c) Calculate the gross profit margin for the year.

Solution 179 (20–25 min.)


(a) Perpetual
RUMMY LTD.
Income Statement (partial)
Year Ended December 31, 2015
___________________________________________________________________________
Sales.................................................................................................... $738,000
Less: Sales returns and allowances..................................................... $28,000
Sales discounts.............................................................................. 4,000 32,000
Net sales.............................................................................................. 706,000
Cost of goods sold*.............................................................................. 403,200
Gross profit........................................................................................... $302,800

* $420,000 – $16,800 = $403,200

(b) Periodic
RUMMY LTD.
Income Statement (partial)
Year Ended December 31, 2015
Sales.................................................................................................... $738,000
Less: Sales returns and allowances................................................... $ 28,000

Copyright © 2014 John Wiley & Sons Canada, Ltd. Unauthorized copying, distribution, or transmission of this page is prohibited

Downloaded by Rnz Rnz (renzalconera@gmail.com)


lOMoARcPSD|4061633

5 - 45 Merchandising Operations

Sales discounts......................................................................... 4,000 32,000


Net sales.............................................................................................. 706,000
Cost of goods sold
Inventory, January 1............................................................................. $ 45,000
Purchases............................................................................................ 400,000
Less: Purchases returns and allowances.........................$20,000
Purchase discounts................................................. 6,000 26,000
Net purchases...................................................................................... 374,000
Add: Freight in................................................................................... 10,000
Cost of goods purchased...................................................................... 384,000
Cost of goods available for sale............................................................ 429,000
Inventory, December 31*...................................................................... 25,800
Cost of goods sold................................................................................ 403,200
Gross profit........................................................................................... $302,800

*Since cost of goods sold is the same as under the perpetual system, ending inventory must be
$429,000 – $403,200 = $25,800.

(c) Gross profit margin = $302,800 ÷ $706,000 = 42.9%

*Ex. 180
Below is a series of cost of goods sold sections for four companies that use a periodic inventory
system (in thousands):
Co. A Co. B Co. C Co. D
Beginning inventory (a) 35 12 (m)
Purchases 123 (e) 67 (n)
Purchase returns and allowances (b) 9 (i) 11
Net purchases 113 205 66 178
Freight in (c) 20 (j) 12
Freight out 10 12 9 8
Cost of goods purchased 147 (f) 73 (o)
Cost of goods available for sale 171 (g) (k) 190
Ending inventory (d) (h) 8 (p)
Cost of goods sold 141 235 (l) 171

Instructions
What are the amounts that should appear in the table where a letter in parentheses is shown?

*Solution 180 (15–20 min.)


($ in thousands) Co. A Co. B Co. C Co. D
Beginning inventory $ 24 $ 35 $12 $ 0
Purchases 123 214 67 189
Purchase returns and allowances 10 9 1 11
Net purchases 113 205 66 178
Freight in 34 20 7 12
Freight out 10 12 9 8
Cost of goods purchased 147 225 73 190
Cost of goods available for sale 171 260 85 190

Copyright © 2014 John Wiley & Sons Canada, Ltd. Unauthorized copying, distribution, or transmission of this page is prohibited

Downloaded by Rnz Rnz (renzalconera@gmail.com)


lOMoARcPSD|4061633

5 - 46 Test Bank for Financial Accounting: Tools for Business Decision-Making, 6th Canadian Edition

Ending inventory 30 25 8 19
Cost of goods sold 141 235 77 171

*Ex. 181
On June 1, Charles Charcoal Ltd. had an inventory of 10 barbeques at a cost of $220 each.
Charles uses a periodic inventory system. During the month of June the following transactions
occurred:
Jun 3 Purchased 25 barbeques at a cost of $220 each from Mr BBQ Ltd., terms n/30.
5 Paid $100 freight for the barbeques purchased on June 3.
6 Sold 12 barbeques to Grills Plus More for $380 each, terms 2/10, n/30.
7 Received credit from Mr BBQ for the return of two defective barbeques.
13 Issued a credit to Grills Plus More for the return of one defective barbeque.
16 Received a credit from Mr BBQ for the defective barbeque returned by Grills Plus
More.
19 Purchased 10 barbeques from Holiday Barbeques at a cost of $220 each, terms
2/10, n/30.
20 Paid freight of $100 on the June 19 purchase.

On June 30, Charles’ ending inventory was $3,220.

Instructions
(a) Prepare journal entries to record the above transactions.
(b) Calculate the cost of goods sold for June.

*Solution 181 (20 min.)


(a)
May 3 Purchases ($220 x 25)...................................................... 5,500
Accounts Payable........................................................ 5,500

5 Freight In............................................................................ 100


Cash............................................................................ 100

6 Accounts Receivable ($380 x 12)....................................... 4,560


Sales........................................................................... 4,560

7 Accounts Payable ($220 x 2).............................................. 440


Purchase Returns and Allowances.............................. 440

13 Sales Returns and Allowances........................................... 380


Accounts Receivable................................................... 380

16 Accounts Payable............................................................... 220


Purchase Returns and Allowances.............................. 220

19 Purchases ($220 x 10)....................................................... 2,200


Accounts Payable........................................................ 2,200

20 Freight In............................................................................ 100

Copyright © 2014 John Wiley & Sons Canada, Ltd. Unauthorized copying, distribution, or transmission of this page is prohibited

Downloaded by Rnz Rnz (renzalconera@gmail.com)


lOMoARcPSD|4061633

5 - 47 Merchandising Operations

Cash............................................................................ 100

(b)
Inventory, June 1 (10 @ $220)...................................................... $2,200
Purchases (35 @ $220)................................................................. $7,700
Less: purchase returns and allowances (3 @ $220)...................... 660
Net purchases............................................................................... 7,040
Add: freight in ($100 + $100)......................................................... 200
Cost of goods purchased............................................................... 7,240
Cost of goods available for sale..................................................... 9,440
Inventory, June 30......................................................................... 3,220
Cost of goods sold......................................................................... $6,220

*Ex. 182
Magnesium Inc. uses a periodic inventory system. During April, the following transactions
occurred:
Apr 3 Purchased $2,000 of merchandise, terms 3/10, n/60.
6 Returned $300 of the merchandise purchased on April 3.
7 Paid freight charges of $150 on goods purchased on April 3.
12 Paid for the goods purchased on April 3.
13 Sold goods on credit for $1,000, terms 2/10, n/45.
14 The customer of April 13 returned $300 of the goods.
23 Received payment from the customer of April 13.

Instructions
Prepare journal entries to record the above transactions.

*Solution 182 (20 min.)


Apr 3 Purchases.......................................................................... 2,000
Accounts Payable........................................................ 2,000

6 Accounts Payable............................................................... 300


Purchase Returns and Allowances.............................. 300

7 Freight In............................................................................ 150


Cash............................................................................ 150

12 Accounts Payable ($2,000 – $300)..................................... 1,700


Purchase Discounts ($1,700 x 3%)............................. 51
Cash ($1,700 x 97%).................................................. 1,649

13 Accounts Receivable.......................................................... 1,000


Sales........................................................................... 1,000

14 Sales Returns and Allowances........................................... 300


Accounts Receivable................................................... 300

23 Cash ($700 x 98%)............................................................. 686


Sales Discounts ($700 x 2%)............................................. 14

Copyright © 2014 John Wiley & Sons Canada, Ltd. Unauthorized copying, distribution, or transmission of this page is prohibited

Downloaded by Rnz Rnz (renzalconera@gmail.com)


lOMoARcPSD|4061633

5 - 48 Test Bank for Financial Accounting: Tools for Business Decision-Making, 6th Canadian Edition

Accounts Receivable ($1,000 – $300)......................... 700

*Ex. 183
Pacific Supply Corporation uses a periodic inventory system. During September, the following
transactions occurred:
Sep 3 Purchased 36 backpacks at $25 each from Scott Limited, terms 2/10, n/30.
6 Received credit of $100 for the return of 4 backpacks purchased on Sept. 3 that
were defective.
9 Sold 20 backpacks for $45 each to Macklin Books, terms 2/10, n/30.
13 Paid Scott account in full.

Instructions
Prepare journal entries to record the above transactions.

*Solution 183 (15 min.)


Sep 3 Purchases ($25 x 36)......................................................... 900
Accounts Payable........................................................ 900

6 Accounts Payable............................................................... 100


Purchase Returns and Allowances.............................. 100

9 Accounts Receivable ($45 x 20)......................................... 900


Sales........................................................................... 900

13 Accounts Payable ($900 – $100)....................................... 800


Purchase Discounts ($800 × 2%)................................ 16
Cash ($800 x 98%)..................................................... 784

*Ex. 184
Babylon Corporation uses a periodic inventory system. During October, the following
transactions occurred:
Oct 3 Purchased $16,000 of merchandise on credit, terms 4/10, n/30.
6 Returned $1,600 of the goods purchased on Oct 3.
7 Paid freight charges of $250 for goods purchased on Oct 3.
12 Paid for the goods purchased on Oct 3.

Instructions
Prepare journal entries to record the above transactions.

*Solution 184 (15 min.)


Oct 3 Purchases..........................................................................16,000
Accounts Payable........................................................ 16,000

6 Accounts Payable............................................................... 1,600


Purchase Returns and Allowances.............................. 1,600

7 Freight In............................................................................ 250

Copyright © 2014 John Wiley & Sons Canada, Ltd. Unauthorized copying, distribution, or transmission of this page is prohibited

Downloaded by Rnz Rnz (renzalconera@gmail.com)


lOMoARcPSD|4061633

5 - 49 Merchandising Operations

Cash............................................................................ 250

12 Accounts Payable ($16,000 – $1,600)................................ 14,400


Purchase Discounts ($14,400x 4%)............................ 576
Cash ($14,400 x 96%)................................................. 13,824

*Ex. 185
The most recent income statement of Lawerence Limited includes the items listed below:
Beginning inventory..................................................... $ 900,000
Freight in..................................................................... 20,000
Gross profit.................................................................. 1,400,000
Net sales..................................................................... 3,750,000
Operating expenses.................................................... 300,000
Purchases................................................................... 1,520,000
Purchase discounts..................................................... 35,000
Purchase returns and allowances................................ 12,000

Instructions
Use the appropriate items listed above as a basis for calculating:
(a) Cost of goods sold.
(b) Cost of goods available for sale.
(c) Ending inventory.

*Solution 185 (15 min.)


(a) Net sales – Cost of goods sold = Gross profit
$3,750,000 – Cost of goods sold = $1,400,000
Cost of goods sold = $2,350,000

(b) Beginning inventory..................................................... $ 900,000


Purchases................................................................... $1,520,000
Less: Purchase discounts............................................ $35,000
Purchase returns and allowances...................... 12,000 47,000
Net Purchases............................................................. 1,473,000
Add: Freight in............................................................ 20,000
Cost of goods purchased............................................. 1,493,000
Cost of goods available for sale................................... $2,393,000

(c) Cost of goods available for sale – Ending inventory = Cost of goods sold
$2,393,000 – Ending inventory = $2,350,000
Ending inventory = $43,000

*Ex. 186
Given the following information, prepare in good form the cost of goods sold section of an
income statement, using the periodic inventory system.
Beginning inventory..................................................... $15,000
Ending inventory.......................................................... 16,000

Copyright © 2014 John Wiley & Sons Canada, Ltd. Unauthorized copying, distribution, or transmission of this page is prohibited

Downloaded by Rnz Rnz (renzalconera@gmail.com)


lOMoARcPSD|4061633

5 - 50 Test Bank for Financial Accounting: Tools for Business Decision-Making, 6th Canadian Edition

Freight in..................................................................... 4,000


Purchases................................................................... 38,000
Purchase discounts..................................................... 500
Purchase returns and allowances................................ 1,800

*Solution 186 (15 min.)


Beginning inventory............................................................ $15,000
Purchases.......................................................................... $38,000
Less: Purchase returns and allowances............................ $1,800
Purchase discounts.................................................. 500 2,300
Net purchases.................................................................... 35,700
Freight in............................................................................ 4,000
Cost of goods purchased.................................................... 39,700
Cost of goods available for sale......................................... 54,700
Ending inventory................................................................. 16,000
Cost of goods sold.............................................................. $38,700

*Ex. 187
Three items are missing in each of the following columns and are identified by a letter.
Sales $ (a) $860,000
Sales returns and allowances 15,000 20,000
Sales discounts 10,000 15,000
Net sales 450,000 (d)
Beginning inventory (b) 325,000
Cost of goods purchased 200,000 (e)
Ending inventory 170,000 303,000
Cost of goods sold 250,000 575,000
Gross profit (c) (f)

Instructions
Calculate the missing amounts and identify them by letter.

*Solution 187 (15 min.)


(a) $475,000

(b) $220,000

(c) $200,000

(d) $825,000

(e) $553,000

(f) $250,000

*Ex. 188
Mendez Electronics Limited uses the periodic inventory system and prepares monthly financial

Copyright © 2014 John Wiley & Sons Canada, Ltd. Unauthorized copying, distribution, or transmission of this page is prohibited

Downloaded by Rnz Rnz (renzalconera@gmail.com)


lOMoARcPSD|4061633

5 - 51 Merchandising Operations

statements. All accounts have been adjusted except for merchandise inventory. A physical
count of merchandise inventory on September 30, 2015 indicates that $2,000 was on hand. A
partial listing of adjusted account balances follows:
Accounts payable........................................................ $ 7,250
Accounts receivable.................................................... 8,000
Cash............................................................................ 22,000
Freight in..................................................................... 1,100
Income tax expense.................................................... 1,530
Merchandise inventory, September 1 ......................... 1,500
Operating expenses.................................................... 23,100
Purchases................................................................... 35,000
Purchase returns and allowances................................ 350
Sales........................................................................... 70,000
Sales discounts........................................................... 750

Instructions
Prepare a multiple-step income statement for Hernandez Book Store for the month ended
September 30, 2015.

*Solution 188 (15 min.)


MENDEZ ELECTRONICS LIMITED
Income Statement
Month Ended September 30, 2015
___________________________________________________________________________
Sales revenues
Sales........................................................................... $70,000
Less: Sales discounts.................................................. 750
Net sales..................................................................... $69,250

Cost of goods sold


Merchandise inventory, September 1.......................... $ 1,500
Purchases................................................................... $35,000
Less: Purchase returns and allowances...................... 350
Net purchases............................................................. 35,350
Add: Freight in............................................................. 1,100
Cost of goods purchased............................................. 36,450
Cost of goods available for sale................................... 37,950
Merchandise inventory, September 30........................ 2,000
Cost of goods sold................................................ 35,950
Gross profit......................................................................... 33,300
Operating expenses........................................................... 23,100
Profit before income tax..................................................... 10,200
Income tax expense........................................................... 1,530
Profit................................................................................... $ 8,670

Copyright © 2014 John Wiley & Sons Canada, Ltd. Unauthorized copying, distribution, or transmission of this page is prohibited

Downloaded by Rnz Rnz (renzalconera@gmail.com)


lOMoARcPSD|4061633

5 - 52 Test Bank for Financial Accounting: Tools for Business Decision-Making, 6th Canadian Edition

MATCHING QUESTIONS

189. Match the items below by entering the appropriate code letter in the space provided.

A. Net sales F. Contra revenue


B. Sales discount G. Freight out
C. Credit terms H. Gross profit
D. Periodic inventory system I. Sales invoice
E. Gross profit margin J. Purchase discount

_____ 1. A reduction given by the seller for prompt payment of a credit sale

_____ 2. Provides support for a credit sale

_____ 3. Gross profit divided by net sales

_____ 4. Sales less sales returns and allowances and sales discounts

_____ 5. Specifies the amount of cash discount and time period during which it is offered.

_____ 6. Net sales less cost of goods sold

_____ 7. Freight cost to deliver goods to customers reported as an operating expense.

_____ 8. Requires a physical count of goods on hand to calculate cost of goods sold.

_____ 9. A cash discount claimed by a buyer for prompt payment of a balance due.

_____ 10. An account that is offset against a revenue account on the income statement.

Copyright © 2014 John Wiley & Sons Canada, Ltd. Unauthorized copying, distribution, or transmission of this page is prohibited

Downloaded by Rnz Rnz (renzalconera@gmail.com)


lOMoARcPSD|4061633

5 - 53 Merchandising Operations

ANSWERS TO MATCHING QUESTIONS

1. B

2. I

3. E

4. A

5. C

6. H

7. G

8. D

9. J

10. F

Copyright © 2014 John Wiley & Sons Canada, Ltd. Unauthorized copying, distribution, or transmission of this page is prohibited

Downloaded by Rnz Rnz (renzalconera@gmail.com)


lOMoARcPSD|4061633

5 - 54 Test Bank for Financial Accounting: Tools for Business Decision-Making, 6th Canadian Edition

SHORT-ANSWER ESSAY QUESTIONS

S-A E 190
Describe the types of inventories that organizations may report on their statements of financial
position. What kind of businesses would report what type of inventory?

Solution 190
Retailers and wholesalers would report merchandise inventory, which is in a form ready to sell
to customers (e.g., Walmart, Loblaw, etc.).

Manufacturers would report raw materials inventory (basic materials on hand ready to go into
production), work in process inventory (inventory which has been started into production but
is not yet complete), and finished goods inventory (manufactured items that are complete and
ready for sale).

S-A E 191
The periodic and the perpetual inventory systems are two methods that companies use to
account for inventories. Briefly describe the major features of each system and explain why a
physical inventory is necessary under both systems.

Solution 191
When a periodic inventory system is used, the Inventory account remains the same throughout
the period. Separate accounts, such as Purchases, Freight In, and Purchase Discounts, are
used to record the transactions. Cost of goods sold is determined by the following formula:
Beginning inventory + Purchases – Ending inventory.

The determination of ending inventory is made by a physical count.

When a perpetual inventory system is used, the purchase and sale of goods are recorded
directly in the Inventory account, which eliminates the need for separate accounts. Cost of
goods sold is recognized for each sale by debiting Cost of Good Sold and crediting Inventory. At
the end of the period, the ending account balance should equal inventory's ending balance.
However, a company should conduct a physical inventory count at least once a year, because
there could be differences resulting from spoilage, theft, or errors.

S-A E 192
What is the main consideration when choosing between a periodic and a perpetual inventory
system?

Solution 192
When choosing between a periodic and perpetual inventory system, a company should consider
the additional costs associated with keeping detailed inventory records versus the benefits of
having additional information about, and control over their inventory.

S-A E 193
Distinguish between cost of goods sold, operating expenses, and non-operating expenses.

Copyright © 2014 John Wiley & Sons Canada, Ltd. Unauthorized copying, distribution, or transmission of this page is prohibited

Downloaded by Rnz Rnz (renzalconera@gmail.com)


lOMoARcPSD|4061633

5 - 55 Merchandising Operations

Describe the nature of these three items and their placement on a multiple-step income
statement.

Solution 193
Cost of goods sold includes the cost of obtaining the goods held for resale; it is deducted
directly from net sales on the income statement. Net sales less cost of goods sold results in
gross profit. Operating expenses, on the other hand, appear directly below the gross profit on
the income statement. Operating expenses include the costs of running the day-to-day
operations of the business such as rent, salaries and insurance. Non-operating expenses are
expenses unrelated to daily operations, such as interest expense.

S-A E 194
The income statement for a merchandising company presents three amounts not shown in a
service company’s income statement. Identify and briefly explain the three unique amounts.

Solution 194
The items reported for a merchandising company that are not reported for a service company
are: sales revenues, cost of goods sold, and gross profit. Sales revenues consist of sales, sales
returns and allowances, and sales discounts. Cost of goods sold represents the total cost of
merchandise sold during the period. Gross profit is the excess of net sales over the cost of
goods sold.

S-A E 195
Public companies in Canada must list expenses on the income statement either by nature or
function. Explain what this means. Are private companies required to do the same?

Solution 195
Classifying expenses by nature means that expenses are reported according to their natural
classifications, e.g., salaries, depreciation, advertising, utilities. Classifying expenses by
function means that expenses are reported according to the activity (business function) for
which they were incurred, e.g., cost of goods sold, administration, selling expenses. An
organization has the choice to classify by nature or by function – the choice should be based on
whichever provides more relevant information. Note expenses may be listed in any order within
the chosen classification. Note also that organizations following ASPE may list their expenses in
whatever order they choose, or they may list by nature or function.

S-A E 196
In a single-step income statement, all data (except for income tax) are classified under two
categories: (1) Revenues, or (2) Expenses. If the income statement is recast in a multiple-step
format, what additional information or intermediate components of revenue would be presented?

Solution 196
The items reported in a multiple-step income statement that are not reported in a single-step
income statement are gross revenues as well as net revenues, cost of goods sold, gross profit,
operating expenses, profit from operations, other revenues and gains, and other expenses and
losses.

Copyright © 2014 John Wiley & Sons Canada, Ltd. Unauthorized copying, distribution, or transmission of this page is prohibited

Downloaded by Rnz Rnz (renzalconera@gmail.com)


lOMoARcPSD|4061633

5 - 56 Test Bank for Financial Accounting: Tools for Business Decision-Making, 6th Canadian Edition

S-A E 197
You are working for the summer at PLC Ltd., a company that operates a chain of retail stores. In
past, the company has not disclosed its cost of goods sold, but now is required to do so. The
company president would like to know the pros and cons of disclosing information. Prepare a
memo to the president containing the information requested.

Solution 197
MEMO
TO: President, PLC Ltd.
FROM: Accounting Student
RE: Disclosure of cost of goods sold
DATE: June xx, xxxx

Disclosing the cost of goods sold enables users of the statements to better
evaluate the company’s performance. They can see the relationship between the
company’s sales and its cost of goods sold. The downside of disclosing the
information is that competitors can also have access to this information. For
example, they can use the information to estimate the company’s mark-up
although its value will be limited as they can only calculate the mark-up in its
aggregate (total) and not by product category.

S-A E 198
You are working as an accounting clerk for Jakubo Wholesalers for the summer. You notice that
some invoices that look like inventory purchases are debited to the Operating Expenses
account. When you ask your supervisor about the invoices, she says you don’t need to be
concerned about it because it won’t have any effect on the profit.

Instructions
Does the classification of the invoices matter? Explain.

Solution 198
Classifying the invoices as operating expenses rather than inventory will have the immediate
effect of understating current assets on the statement of financial position and cost of goods
sold on the income statement. Subsequently, when the inventory is sold in a later period, cost of
goods sold will be understated and gross profit overstated.

Not properly distinguishing on the income statement between cost of goods sold and operating
expenses will increase the gross profit and gross profit margin. The gross margin is important in
evaluating the company’s performance, so the misclassification does matter.

S-A E 199
Explain why gross profit margin is considered to be more informative than gross profit.

Solution 199
Gross profit margin expresses a more meaningful relationship between gross profit and sales.
Specifically, it shows how much gross profit a company earns for each $1 in net sales it
generates. This puts gross profit into perspective and draws attention to a company’s

Copyright © 2014 John Wiley & Sons Canada, Ltd. Unauthorized copying, distribution, or transmission of this page is prohibited

Downloaded by Rnz Rnz (renzalconera@gmail.com)


lOMoARcPSD|4061633

5 - 57 Merchandising Operations

profitability relative to its size.

*S-A E 200
A merchandising company using the periodic system frequently has the need to use contra
accounts related to the purchase and sale of goods. Identify the contra accounts that have (1)
normal credit balances and explain why they are not considered revenues, and (2) normal debit
balances and explain why they are not considered expenses.

*Solution 200
1. The contra accounts related to the purchase of goods that have normal credit balances are
Purchase Discounts and Purchase Returns and Allowances. These accounts have credit
balances because they are adjustments to purchases, not revenues. They are an
adjustment of the outflow from the purchase of goods, rather than a revenue generating
activity.

2. The contra accounts related to the sale of goods that have normal debit balances are Sales
Discounts and Sales Returns and Allowances. These accounts have debit balances but are
not expenses because they are adjustments of sales, not operating, selling, or
administrative expenses. They are an adjustment of the inflow from sale of goods, rather
than a cost used to help earn revenue.

Copyright © 2014 John Wiley & Sons Canada, Ltd. Unauthorized copying, distribution, or transmission of this page is prohibited

Downloaded by Rnz Rnz (renzalconera@gmail.com)


lOMoARcPSD|4061633

5 - 58 Test Bank for Financial Accounting: Tools for Business Decision-Making, 6th Canadian Edition

LEGAL NOTICE

Copyright © 2014 by John Wiley & Sons Canada, Ltd. or related companies. All rights
reserved.

The data contained in these files are protected by copyright. This manual is furnished
under licence and may be used only in accordance with the terms of such licence.

The material provided herein may not be downloaded, reproduced, stored in a retrieval
system, modified, made available on a network, used to create derivative works, or
transmitted in any form or by any means, electronic, mechanical, photocopying,
recording, scanning, or otherwise without the prior written permission of John Wiley &
Sons Canada, Ltd.

Copyright © 2014 John Wiley & Sons Canada, Ltd. Unauthorized copying, distribution, or transmission of this page is prohibited

Downloaded by Rnz Rnz (renzalconera@gmail.com)

Das könnte Ihnen auch gefallen